You are on page 1of 118

MUT DEPARTMENT OF MATHEMATICAL SCIENCES

Mathematics 1 Tutorial
Guide

Mr D. Hove
1/19/2015

This guide is the Tutorial Guide for Mathematics 1, containing additional revision material,
tutorial questions as well as some foundation material required for the course. It closely
follows and is informed by the Mathematics 1 study guide which is available on Blackboard.
Contents
MATHEMATICS 1 TUTORIAL POLICY ............................................................................... 5
EXPLANATION OF ICONS USED ......................................................................................... 6
MATHEMATICS 1 OVERVIEW ............................................................................................. 7
CONTENT AREA 1: ALGEBRA ............................................................................................. 8
1 Conic Sections .................................................................................................................... 9
1.1 Learning Outcomes ..................................................................................................... 9
1.2 Introduction ................................................................................................................. 9
1.3 Ellipse .......................................................................................................................... 9
Learning Activity: Remembering ellipses ........................................................................ 10
Learning Activity: Two worked examples ....................................................................... 10
Worked Example 1: .......................................................................................................... 11
Solution and Reflection .................................................................................................... 11
Worked Example 2: .......................................................................................................... 12
Solution and Reflection .................................................................................................... 12
1.4 Self-study .................................................................................................................. 13
1.5 Tutorial 1 ................................................................................................................... 13
Reflection and comment ................................................................................................... 13
2 Exponential and Logarithmic Functions........................................................................... 15
2.1 Learning Outcomes ................................................................................................... 15
2.2 Introduction ............................................................................................................... 15
2.3 Laws of Indices ......................................................................................................... 15
2.4 Exponential Functions ............................................................................................... 15
Base 𝒆 ............................................................................................................................... 16
Laws of indices for base e ................................................................................................ 16
2.5 Laws of logarithms (revision) ................................................................................... 16
Natural logarithm .............................................................................................................. 17
Learning Activity: A Simplification Example.................................................................. 18
Simplification Example 1: ................................................................................................ 18
Solution and Reflection .................................................................................................... 18
2.6 Self Study .................................................................................................................. 19
2.7 Revision Exercises .................................................................................................... 21
3 Exponential and Logarithmic Equations .......................................................................... 22
3.1 Learning Outcomes ................................................................................................... 22
3.2 Introduction ............................................................................................................... 22
3.3 Quadratic Equations (Revision) ................................................................................ 22
Learning Activity: Remembering Quadratic Equations ................................................... 22

1
3.4 Quadratic Equations (Revision) ................................................................................ 24
Learning Activity 3.4.1: Remembering Quadratic Equations .......................................... 24
3.5 Self Study .................................................................................................................. 27
4 Applications: Exponential Growth and Decay ................................................................. 29
4.1 Learning Outcomes ................................................................................................... 29
4.2 Introduction ............................................................................................................... 29
Learning Activity 4.2.1: Solving Exponential Growth and Decay Equations.................. 29
Learning Activity 4.2.2: Growth and Decay examples .................................................... 31
Rounding off ..................................................................................................................... 34
4.3 Self Study .................................................................................................................. 35
5 Radian Measure ................................................................................................................ 36
5.1 Introduction ............................................................................................................... 36
5.2 Radian Measure ......................................................................................................... 36
Length of an arc ................................................................................................................ 37
Area of a Sector of a circle ............................................................................................... 37
Area of a triangle .............................................................................................................. 38
Area of a segment ............................................................................................................. 38
6 The Binomial Theorem ..................................................................................................... 40
CONTENT AREA 2: TRIGONOMETRY .............................................................................. 46
7 Trigonometric Graphs....................................................................................................... 47
7.1 The properties of a sine (cosine) wave ...................................................................... 47
The value of ‘quarter points’ ............................................................................................ 49
Sketching the sine function............................................................................................... 49
8 Trigonometric Graphs and Equations ............................................................................... 52
8.1 Solving trigonometric equation graphically .............................................................. 52
8.2 General Solution of Trigonometric Equations .......................................................... 54
8.3 Test 1 ......................................................................................................................... 57
CONTENT AREA 3: DIFFERENTIAL CALCULUS............................................................ 58
9 Differentiation – Basic...................................................................................................... 59
9.1 Introduction to differentiation ................................................................................... 59
Differentiation Notation ................................................................................................... 59
9.2 Differentiating 𝒙𝒏 with respect to 𝒙 ......................................................................... 60
9.3 Differentiating a constant .......................................................................................... 61
9.4 Differentiating a linear function of x......................................................................... 61
9.5 Other common derivatives ........................................................................................ 62
Derivative of 𝒚 = 𝐬𝐢𝐧𝒙 .................................................................................................... 62
Derivative of 𝒚 = 𝐜𝐨𝐬𝒙 .................................................................................................... 62

2
Derivative of 𝒚 = 𝐭𝐚𝐧𝒙 .................................................................................................... 62
Derivative of 𝒚 = 𝒆𝒙 ........................................................................................................ 62
Derivative of 𝒚 = 𝐥𝐧𝒙 ...................................................................................................... 62
10 Differentiation of Compound Functions ....................................................................... 64
10.1 Differentiation of a product of functions................................................................... 64
10.2 Differentiation a quotient .......................................................................................... 66
10.3 Differentiating a function of a function..................................................................... 66
10.4 Use of standard derivatives formulae ........................................................................ 69
Derivative of a function of a function of a function of a function…................................ 71
11 Differentiating Implicit Functions ................................................................................ 76
11.1 Implicit Functions ..................................................................................................... 76
Derivative of an Implicit Function ................................................................................... 76
11.2 Logarithmic Differentiation ...................................................................................... 79
12 Applications of Differentiation ..................................................................................... 83
12.1 Finding the gradient at a particular point on a curve................................................. 83
12.2 Determining the equations of a tangent and a normal ............................................... 83
12.3 Higher Order Derivatives .......................................................................................... 84
12.4 Stationary Values ...................................................................................................... 85
12.5 Displacement, velocity, acceleration: Linear Motion ............................................... 87
12.6 Displacement, velocity and acceleration: Circular Motion ....................................... 87
CONTENT AREA 4: INTEGRAL CALCULUS .................................................................... 89
13 Basic Integration ........................................................................................................... 90
13.1 Differentiation Revisited ........................................................................................... 90
The Constant of Integration .............................................................................................. 90
13.2 Integrating a Sum or Difference of Functions ........................................................... 91
13.3 Integrating 𝒂𝒙 + 𝒃𝒏 .................................................................................................. 91
13.4 Standard Integrals ...................................................................................................... 93
13.5 Integrating Exponential Functions ............................................................................ 93
13.6 How to Integrate 𝟏𝒙 .................................................................................................. 95
13.7 Integrating Trigonometric Functions ........................................................................ 97
13.8 Integration Functions of the Form 𝒇′(𝒙)𝒇𝒙𝒏............................................................ 98
14 Definite Integrals ........................................................................................................ 101
14.1 Definite Integration ................................................................................................. 101
Finding area under a curve by definite integration ......................................................... 102
CONTENT AREA 5: COMPLEX NUMBERS .................................................................... 105
15 Complex Numbers: Basic Definitions; Operations and Graphical Representation .... 106
15.1 Introduction ............................................................................................................. 106

3
15.2 The imaginary number 𝒋 .......................................................................................... 106
Powers of 𝒋...................................................................................................................... 107
15.3 Complex Numbers................................................................................................... 108
Addition and Subtraction of Complex Numbers ............................................................ 108
Multiplication of Complex Numbers .............................................................................. 108
Division of Complex Numbers ....................................................................................... 109
Equality of Complex Numbers ....................................................................................... 109
15.4 Graphical representation of Complex Numbers ...................................................... 111
16 Complex Numbers: Polar, Exponential Forms and De Moivre’s Theorem................ 112
16.1 Polar and Exponential Form of a Complex Number ............................................... 112
16.2 Multiplication and Division of Complex Numbers in Polar Form ......................... 114
16.3 De Moivre’s Theorem ............................................................................................. 115

4
MATHEMATICS 1 TUTORIAL POLICY
• Mathematics 1 tutorial sessions will be conducted every Friday afternoon. The first
session will be from 1300 to 1430 hrs and the second session will be from 1430 to
1600 hrs.
• Students should attend all the tutorial sessions to which they have been allocated.
Students who do not attend sessions to which they have been allocated will be
regarded as absent. Refer to rules G. 16 (timetable clashes) and G. 21 (absence from
lectures and tests).
• Before attending a particular session, students should attempt all the tutorial problems
to be discussed in that particular tutorial session.
• Tutorial tests will be written at the end of some tutorial sessions and test dates may
not be announced in advance.
• Each tutorial test will be based on the tutorial for the day.
• There will be no re-tests or make-up tests. Students who have missed any of the
tutorial tests will have to see their respective Mathematics lecturers.
• The average of the tutorial tests will constitute a third score for the semester and this
score will in turn constitute 20% of the semester course mark.
• You will write 2 tutorial assignments and 1 tutorial test.
• The tutorial mark is calculated as follows:

Tutorial Mark = 0.5*(Assignment 1 +Assignment 2) + 0.5*Tutorial Test

• On top of that, you write 2 semester tests. You calculate your DP as follows:

=DP 0.4*(Test 1+Test 2) + 0.2*Tutorial Mark

5
EXPLANATION OF ICONS USED

ICON Explanation
Learning Activity

Reflection and Solution

Self-study

6
MATHEMATICS 1 OVERVIEW

MATHEMATICS 1

Content Area 1: Algebra

Content Area 2: Trigonometry

Content Area 3: Differential Calculus

Content Area 4: Integral Calculus

Content Area 5: Complex Numbers

7
CONTENT AREA 1: ALGEBRA

ALGEBRA
Conic Sections

Exponential Functions with base e

Natural Logarithmic Functions

Exponential Equations

Logarithmic Equations

Mathematical Modeling Involving Exponential Decay


and Growth

Radian Measure

The Binomial Theorem

8
1 Conic Sections
1.1 Learning Outcomes
After completing this section, you should be able to
• Sketch the graph of an ellipse with the centre at the origin
• Find the standard form of the equation of an ellipse with the centre at the origin
• Identify vertices, major axis, minor axis, foci and use these to find the equation of an
ellipse

1.2 Introduction
From your lectures, you know that an ellipse is part of a family of curves called conic
sections. Other members of this family include the hyperbola, parabola, circle, etc. However,
for purposes of the Mathematics 1 module, the only conic section to be analysed is an ellipse
with its centre at the origin.

Let’s remind ourselves about the forms of ellipses.

1.3 Ellipse
When centred at the origin, the ellipse is in either of the 2 forms described below.

a. Major axis along the x-axis

Standard form of ellipse for (a) above:


x2 y 2
= + 1 (a > b > 0)
a 2 b2
c=
2
a 2 − b2

𝑥 − intercepts ∶ ±𝑎 (vertices)
𝑦 − intercepts: ± 𝑏
Foci: 𝐹 ′ (−𝑐, 0), 𝐹(𝑐, 0)
Major axis length = 2𝑎

9
Minor axis length = 2𝑏

b. Major axis along the y-axis

Standard form of ellipse with major axis along the y-axis


x2 y 2
= + 1 ( a > b > 0)
b2 a 2
c=
2
a 2 − b2

𝑥 − intercepts: ± b
𝑦 − intercepts: ± a (vertices)
Foci: 𝐹 ′ (0, −𝑐), 𝐹(0, 𝑐)
Major axis length = 2𝑎
Minor axis length = 2𝑏
Learning Activity: Remembering ellipses

1. Look at the two forms of ellipses, above, again carefully.


2. In each case, what do the letters a and b represent?
3. What is different about the x and y intercepts in the two forms?
4. What is the same and different about the foci in each case?

Learning Activity: Two worked examples


Now, let’s test what we understand about ellipses by doing the following worked
examples. These are examples from your study guide, on page 20.

10
Worked Example 1:
Sketch the ellipse indicated by the equation below. State the coordinates of the centre,
vertices and foci.

4𝑥 2 + 9𝑦 2 = 36

Solution and Reflection


The following is a worked solution for Example 1 above. Please don’t just copy the
solution. That will not help you to learn. Try to do the example yourself before you look at
the solution, using what you have learned from the previous learning activities. This will be
much more helpful for your own learning. When you have studied the worked example you
will be able to apply your understanding on your own in the Self Study activities below in
1.4.

Clearly, the equation is not in the standard form of an ellipse. To convert the above equation
into an equation in standard form of an ellipse, simply divide throughout by 36! After
dividing by 36 now we have:

𝑥2 𝑦2
+ =1
9 4
OR

𝑥2 𝑦2
+ =1
32 22

From what we remembered above about the equation for the second form of an ellipse, 𝑎 = 3
and 𝑏 = 2.
To get the x-intercepts, we set y=0, solve and get 𝑥 = ±3 (Show it)
Likewise, to get the y-intercepts, we set x=0, solve and get 𝑦 = ±2
The coordinates of the centre are (0;0) by default.
The vertices are the end-points of the major axis, clearly these are 𝑥 = ±3. However, the
question is asking for coordinates and not just the x-values and so you state the coordinates
properly as (±3; 0) or in expanded form as (−3; 0) and (3; 0).
For the foci, remember 𝑐 2 = 𝑎2 − 𝑏 2 = 32 − 22 = 9 − 4 = 5
Therefore, 𝑐 = √5 and the coordinates of the foci are (�±√5; 0�.
NOTE: Always leave the answer in surd form, where applicable. In other words, do not use a
calculator to get the value of √5

11
Make sure you observe certain things in the diagram above. The only points that I have
labelled are the x- and y-intercepts and the foci.
What about the other points in between like 1, 1.5, etc.? To answer this question, go back to
the question itself, in fact, the very first word of the question. It says sketch, so? For now and
for all the sketching questions which will follow, whenever you are told to sketch something,
only indicate the critical points, no more! All the other points are out of your domain.

However, it is important that at least the diagram is drawn to a consistent scale. The word
sketch does not mean you use free-hand to draw the x- and y-axis, you will get penalised for
recklessness! Always use a ruler or your student ID if a ruler is not immediately available.

Worked Example 2:
Write, in standard form, the equation of the ellipse having the given properties
a) Centre (0;0); foci (±2; 0); vertices (±5; 0)
b) End points of the major and minor axis are (-8;0); (8;0) (0;-4) and (0;4)

Solution and Reflection


Now here is a worked solution for Example 2.

a) Clearly 𝑐 = 2 and 𝑎 = 5 (Note vertices). Therefore


𝑐 2 = 𝑎2 − 𝑏 2
𝑏 2 = 𝑎2 − 𝑐 2 = 52 − 22 = 25 − 4 = 21
∴ 𝑏 = √21

Remember to always leave the answer in surd form, where applicable. In other words, do not
use a calculator to get the value of √21

b) By inspection,=
a 8,=b 4, therefore
x2 y 2
+ = 1
82 4 2
Always stick to the given instruction, do not overshoot a solution.

12
1.4 Self-study
Look back at the intended learning outcomes given at the beginning of this section.
This gives you a review of what you have managed to cover. Now, it’s time for you
to go on and practice the following questions by yourself.

You can compare and check your work with a fellow student. If you disagree on the working,
or on the answer, work together until you are happy to agree. If you are still not sure about
some of your solutions, keep your solutions and discuss them with your tutor in the next
tutorial.

Hopefully this revision and the worked examples have prepared you to be able to do the
exercises in Tutorial 1 independently. Try them, and feel free to compare your working and
your solutions with a fellow student. Raise any questions you have with your tutor.

1.5 Tutorial 1
(page 20 of the study guide. Do these questions before you come for the tutorial.)

Before coming to the tutorial, make sure you do questions 1 and 2 from this tutorial.

Reflection and comment


In this chapter, we have seen that the standard form of the ellipse depends on
which axes the major axis lies. In both cases; the letter a represents the end-point of the major
axis, whereas the letter b represents the end-point of the minor axis.

If the major axis of the ellipse is along the x-axis, we have the following standard form of the
ellipse:
x2 y 2
+ = 1
a 2 b2
c2 = a 2 − b2 , a>b
On the other hand, if the major axis of the ellipse is along the y-axis, we have the following
standard form of the ellipse:
x2 y 2
+ = 1
b2 a 2
c2 = a 2 − b2 , a>b

Therefore, 𝑎 > 𝑏 > 0 AND 𝑐 2 = 𝑎2 − 𝑏 2 , irrespective of which of the two ellipses you are
considering. The term vertices (singular vertex), denotes the end-points of the major axis.
The foci (singular focus) are the two focus points when drawing the ellipse.
When asked to sketch an ellipse, the following are required:
• x-intercept
• y-intercept
• the two foci which should be on the major axis of the ellipse

13
Do not use a calculator to evaluate the value of a, b or c if it comes in surd form. Using a
calculator doesn’t make your solution more exact. In fact, the opposite is true due to round
off errors.

14
2 Exponential and Logarithmic Functions
2.1 Learning Outcomes
After completing this section, you should be able to
• Apply the laws of exponents
• Apply the laws of logarithms

2.2 Introduction
This section begins by reviewing the basic laws of exponents and logarithms. Understanding
of these laws is crucial in all the other areas you will encounter dealing with indices and
logarithms, and that extends beyond this content area. Let’s begin by reviewing these laws
from what you already know from high school.

2.3 Laws of Exponents


The following laws of exponents are true for all the non-zero values of a, m and n:

1. 𝑎𝑚 × 𝑎𝑛 = 𝑎𝑚+𝑛 eg 𝑎3 × 𝑎9 = 𝑎3+9 = 𝑎12


2. 𝑎𝑚 ÷ 𝑎𝑛 = 𝑎𝑚−𝑛 eg 𝑎2 ÷ 𝑎−5 = 𝑎2−(−5) = 𝑎2+5 = 𝑎7
3. (𝑎𝑚 )𝑛 = 𝑎𝑚𝑛 eg (𝑎3 )2 = 𝑎3×2 = 𝑎6

The above 3 laws are the basic laws of exponents and from these; we can generate other
important results:
4. 𝑎0 = 1
1
5. 𝑎−𝑚 = 𝑎𝑚
1
𝑚
6. 𝑎 𝑚 = √𝑎

2.4 Exponential Functions


By definition, an exponential function is one where the unknown variable (x) is in the index.
Examples of exponential functions are 2𝑥 , 3−𝑥 and 10𝑥+1 . Mathematically we say an
exponential function is any function given by 𝑦 = 𝑎 𝑥 , where 𝑎 > 0 and a ≠ 1.

The diagram below shows some members of the exponential family:

15
NOTE:
All the curves pass through the point (1;0). This is because when 𝑥 = 0, 𝑦 = 𝑎 𝑥 = 𝑎0 = 1.
Also, the x-axis is an asymptote for all exponential functions; that is, the exponential
functions approach the x-axis but don’t get to ‘touch’ or ‘cross’ it. We saw 𝑎 𝑥 > 0 for all
values of 𝑥.

Base 𝒆
In this course and in all engineering calculations, the most important exponential is the
exponential function given to base 𝑒. The number 𝑒 is an irrational number with many
important uses in engineering and finance. Please note e is not an abbreviation for
exponential, it is just a symbol. You will get to appreciate its importance as we move along
with the course. The value of 𝑒 from your calculators is 2.718… This base will take over
from base 10 that you were used to in high school. The function 𝑦 = 𝑒 𝑥 has the same shape
as the exponential functions indicated above and its curve would lie between the curves of
𝑦 = 2𝑥 and 𝑦 = 3𝑥 .

Laws of exponents for base e


For the exponential function 𝑒 𝑥 , we have the following laws of exponents
1. 𝑒 𝑚 × 𝑒 𝑛 = 𝑒 𝑚+𝑛 eg 𝑒 2𝑥 × 𝑒 2𝑥+1 = 𝑒 2𝑥+2𝑥+1 = 𝑒 4𝑥+1
𝑒𝑚 𝑒 2𝑥
2. = 𝑒 𝑚−𝑛 eg = 𝑒 2𝑥−(2𝑥−1) = 𝑒 2𝑥−2𝑥+1 = 𝑒 1 = 𝑒
𝑒𝑛 𝑒 2𝑥−1
(𝑒 𝑚 )𝑛 = 𝑒 𝑚𝑛 eg (𝑒 𝑥+1 )𝑥 𝑥 2 +𝑥
3. =𝑒
4. 𝑒 0 = 1 (Any base, except 0, raised to the power of 0 gives 1)
1 1
5. 𝑒 −𝑚 = 𝑒 𝑚 eg 𝑒 −2 = 𝑒 2
1 1
𝑚 4
6. 𝑒 𝑚 = √𝑒 eg 𝑒 4 = √𝑒

2.5 Laws of logarithms (revision)


Logarithm is another word for ‘exponent’. Consider a number like 1 000. You know that
1000 = 10 × 10 × 10 = 103 . To turn the issue around, you can ask, ‘What is the exponent
to which 10 needs to be raised to get 1000?’ The answer in this case is 3. An equivalent
statement is to say, ‘The logarithm of 1000 to base 10 is 3 or log10 1000 = 3’. Logarithms
can also be given in other bases, for example 32 = 2 × 2 × 2 × 2 × 2 = 25 . Therefore, the
logarithm of 32 to base 2 is 5 or log 2 32 = 5. You can add your own examples.
Logarithms used in calculations are normally expressed in base 10. log 1000 = 3 is taken to
mean log10 1000 = 3. As the logarithmic function is the inverse of the exponential function
𝑦 = 𝑎 𝑥 , we can obtain its graph by reflecting about the line 𝑦 = 𝑥, as shown in diagram
below:

16
Note the following:

1. Log 𝑎 1 = 0
2. log 𝑎 𝑥 is not defined if 𝑥 < 0. The logarithm of a negative number does not exist.

The following laws apply to logarithms, and they are rightfully called the laws of logarithms.
You should be able to draw similarities with the laws of indices we have just touched on:

1. log 𝑎 𝑥 + log 𝑎 𝑦 = log 𝑎 𝑥𝑦 Eg. log 𝑎 𝑥 + log 𝑎 (𝑥 + 1) = log 𝑎 𝑥(𝑥 + 1) =


log 𝑎 (𝑥 2 + 𝑥)
𝑥 𝑥
2. log 𝑎 𝑥 − log 𝑎 𝑦 = log 𝑎 𝑦 Eg. log 𝑎 (𝑥) − log 𝑎 (𝑥 − 1) = log 𝑎 𝑥−1
3. log 𝑎 𝑥 𝑦 = 𝑦 log 𝑎 𝑥

These are the 3 basic laws of logarithms. Other important results which follow from this are:

4. log 𝑎 𝑎 = 1
5. log 𝑎 1 = 0
6. log 𝑎 𝑎 𝑥 = 𝑥 Note: 4. and 5 above can be deduced from 6.

Natural logarithm
We have already encountered base e in the discussion for laws of exponents. Likewise, for
logarithms, we take the logarithm to base e. Logarithms to base e are called natural or
Naperian logarithms. The notation for a natural logarithm, which is a logarithm taken to base
e, is ln, that is, log 𝑒 𝑎 = ln 𝑎

Therefore,
ln 𝑎 = 𝑏 ⇔ 𝑎 = 𝑒𝑏

17
The laws of logarithms given above all work perfectly well for natural logarithms, that is:

1. ln 𝑥 + ln 𝑦 = ln 𝑥𝑦
𝑥
2. ln 𝑥 − ln 𝑦 = ln 𝑦
3. ln 𝑥 𝑦 = 𝑦 ln 𝑥
4. ln 𝑒 = 1
5. ln 1 = 0
6. 𝑙𝑛𝑒 𝑥 = 𝑥

Other important results to consider are given below:

ln 𝑥
1. log 𝑎 𝑥 = ln 𝑎
2. ln 𝑒 𝑥 = 𝑒 ln 𝑥 = 𝑥
3. 𝑒 𝑥 ln 𝑎 = 𝑎 𝑥

NOTE: It is very important to know all of these rules and understand them so as to be able to
apply them either in simplification of (expressions involving) exponential and logarithmic
functions and/or in solving exponential and logarithmic equations.

Remember, you simplify an expression and you solve an equation.

Learning Activity: A Simplification Example


Let’s reflect on the laws of indices and (natural) logarithms with a few
simplification examples.
Simplification Example 1:
Question 4
(Page 21 of the study guide)
Simplify the following:
2 𝑒𝑥
4.1 ln(𝑒 3𝑥 ) − �𝑒 ln √𝑥 � − ln �𝑒 𝑥−1 �
𝑒 2𝑥
4.2 ln √𝑒 + √ln 𝑒 − ln 2𝑒 + ln �𝑒 2𝑥−1 �
3 2 2
4.3 �ln 𝑒 𝑥 − �ln √𝑒� �𝑒 ln 4𝑥 � + �𝑒 ln 𝑥 �
3

Remember, try and find your own solutions before you turn the page to read the solution and
reflection.

Solution and Reflection


Now that you have tried the solutions yourself, look at what we have done to find a solution,
and compare it with your own work. Don’t move on until you have clearly understood the
solutions and the process of getting to the solutions.

18
 ex 
(
ln ( e3 x ) − eln ) ( )
2 2
x
− ln  x −1  =−3x x − ln e x − x +1
e 
4.1 = 3 x − x − ln e
= 2x −1

For each of these steps, identify the law of indices or logarithms used. Whilst I have been
careful to do things one at a time, with practice, you will be able to combine some of these
steps.

Let’s now look at the next question. Also take note of the notation here: when asked to
simplify, all the equal signs will be at the left margin and ideally aligned; that makes your
work neat and easy to follow.

 e2 x 
ln e + ln e − ln 2e + ln  2 x −1 = ln e 2 + 1 − ( ln 2 + ln e ) + ln e 2 x − ln e 2 x −1
1

e 
1
= + 1 − ln 2 − 1 + 2 x − ( 2 x − 1)
4.2 2
1
= − ln 2 + 2 x − 2 x + 1
2
3
= − ln 2
2

There’s something you should note here apart from what we have already discussed in the
previous question. It is to do with the format of the final answer. The first part is given as an
improper fraction and the second part is given in form of ln. Always give your fractional
answers as improper fractions where this is required and never, I repeat, never use a
calculator to get the value of ln 2. As already mentioned in chapter 1 on the ellipse, by
using a calculator, you are changing your solution from the exact value to an approximate
value given to a certain number of decimal places.

4.3
3 2 2
�ln 𝑒 𝑥 3 − �ln √𝑒� �𝑒 ln 4𝑥 � + �𝑒 ln 𝑥 �
3 1 2
= �𝑥 3 − �ln 𝑒 2 � (4𝑥) + (𝑥)2
1
= 𝑥 − � � (4𝑥) + 𝑥 2
4
= 𝑥 − 𝑥 + 𝑥2
= 𝑥2

2.6 Self Study


Now, to test your own understanding in the laws of indices/exponents and logarithms, please
go through the following questions before attending the next tutorial. Again, you can
compare and check your work with a fellow student until you both agree on the working, or

19
on the answer. You should also feel free to approach your respective tutors during your
tutorial sessions should you need clarification on the solution or maybe just to double-check.

Question 1
Express the following as a sum or difference of logarithms or as a product
x
a) ln
x −1
b) ln ( 5x 2 )
c) ln ( sin 2 x )
x +1
d) ln
x −1
NB: The knowledge and skill gained in the above question will be ready when we get to
the topic of differentiation. For certain functions, knowledge of the laws of logarithms is
assumed.

Question 2
Express the following as a single logarithm
a) ln x − 2 ln (1 − x )

b) ln sin x − ln cos x
1
c) 4 ln ( x + 1) − ln x
2
1
d) 2 ln x + ln ( x − 1)
2

Word of Caution
In my experience in this course, there are some common mistakes and misconceptions with
regards to the laws of indices and logarithms that I always observe when marking and I feel it
is proper I address them here so that you don’t fall into the same trap. Tutors, please
emphasise these two mistakes in your sessions.

Mistake 1
ln(𝑥 + 𝑦) = ln 𝑥 + ln 𝑦
ln 𝑥(𝑥 + 1) = ln(𝑥 2 + 𝑥) = ln 𝑥 2 + ln 𝑥
ln(𝑥 + 𝑦) = ln(𝑥 + 𝑦) and that’s the end of the story!

20
Mistake 2
ln 𝑎𝑒 𝑥 = 𝑥 ln 𝑎𝑒 never!
The confusion in Mistake 2 comes from the application of this law of logarithms
ln 𝑎 𝑥 = 𝑥 ln 𝑎
To correct the above Mistake 2, what you need to do is this:
ln 𝑎𝑒 𝑥 = ln 𝑎 + ln 𝑒 𝑥 = ln 𝑎 + 𝑥

The above cannot be simplified further.

Other common mistakes:


Mistake 3
ln x ⋅ ln x =
ln x 2

Mistake 4
( ln x ) = 2 ln x
2

2.7 Revision Exercises


Hopefully this revision and the worked examples has prepared you to be able to do the
exercises in Exercise 1 and Exercise 2 (page 15 of the study guide) independently. Try them,
and feel free to compare your working and your solutions with a fellow student. Raise any
questions you have with your tutor.

With what we have learned and applied in this section, it is now time to move on to the next
section where we will learn how to apply the laws of indices and logarithms to solve
exponential and logarithmic equations.

21
3 Exponential and Logarithmic Equations
3.1 Learning Outcomes
After completing this section, you should be able to
• Apply the theory of quadratic equations in solving exponential and logarithmic
equations
• Use a calculator to evaluate expressions involving radical signs, powers of e and
natural logarithms and then round off to the given number of decimal places

3.2 Introduction
In this section, we move on to apply the laws of indices and logarithms to solve exponential
and logarithmic equations. So, make sure you go back and revise those laws in the previous
section and make sure you are fluent in them before proceeding. However, before we delve
into the details of actually solving the exponential and logarithmic equations, let’s touch on
some important topic that is central to this section, the topic on quadratic equations.

3.3 Quadratic Equations (Revision)


Exponential and logarithmic equations encountered in this module normally reduce to
quadratic equations when solving. If you are solving an exponential equation, the exponential
equation reduces to a quadratic equation in terms of e x . Similarly; when solving a
logarithmic equation, the logarithmic equation reduces to a quadratic equation in terms of
ln x. To solve these quadratic equations; you use either factorisation or the quadratic formula
depending on the nature of the question. In the next learning activities, we will revise the
factorisation of quadratic expressions as well as the quadratic formula.

Learning Activity: Remembering Quadratic Equations


There are a number of examples in this activity which will hopefully help you to
remember what you know about quadratic equations. Each example has its own solution and
reflection, but try and do that example on your own, or with a fellow student before you read
the solution and reflection.

Example 1
Factorise 𝑎2 − 17𝑎 + 42

Solution and Reflection

1st step: Find the product of the first and last terms:
𝑎2 × (+42) = +42𝑎2

2nd step: Find two terms such that their sum is -17a and their product is +42𝑎2 . After
checking through the factors through trial and error, the two terms are -14a and -3a.
Therefore, replacing the middle term with these two terms and factorising by grouping, we
have
𝑎2 − 14𝑎 − 3𝑎 + 42 = 𝑎(𝑎 − 14) − 3(𝑎 − 14) = (𝑎 − 3)(𝑎 − 14)

22
The instruction said factorise, not solve. So we leave it here for now and go into the next
example. I would understand if you think that the method I used for factorising was
unnecessary, but there is a reason. Let’s look together at the next example.

Example 2
Factorise 2𝑢2 𝑣 2 + 𝑢𝑣 − 6

Solution and Reflection

Step 1: Find the product of the 1st and last terms:


2𝑢2 𝑣 2 × (−6) = −12𝑢2 𝑣 2

Step 2: As in the previous example, the two terms which have a product of −12𝑢2 𝑣 2 and a
sum of 1𝑢𝑣 are 4𝑢𝑣 and − 3𝑢𝑣.

Therefore,
2𝑢2 𝑣 2 + 4𝑢𝑣 − 3𝑢𝑣 − 6 = 2𝑢𝑣(𝑢𝑣 + 2) − 3(𝑢𝑣 + 2) = (2𝑢𝑣 − 3)(𝑢𝑣 + 2)

Now that you have mastered the factorisation of the quadratic expressions, let’s go on to
solve quadratic equations.

Example 3
Solve the equation 4𝑦 2 + 5𝑦 − 21 = 0.

Solution and Reflection

4𝑦 2 + 5𝑦 − 21 = 0
→ 4𝑦 2 + 12𝑦 − 7𝑦 − 21 = 0
→ 4𝑦(𝑦 + 3) − 7(𝑦 + 3) = 0
→ (4𝑦 − 7)(𝑦 + 3) = 0
→ either (4𝑦 − 7) = 0 or (𝑦 + 3) = 0
7
→ 𝑦= or 𝑦 = −3
4

What if the quadratic expression is not factorisable? For these kinds of questions, we make
use of the formula for solving all quadratic equations, popularly known as the ‘quadratic
formula’. The general form of a quadratic equation is 𝒂𝒙𝟐 + 𝒃𝒙 + 𝒄 = 𝟎. The roots of this
are found by the following formula which is a result of completing the square:

−𝒃 ± √𝒃𝟐 − 𝟒𝒂𝒄
𝒙=
𝟐𝒂

23
Example 4
Find, without using a calculator, the roots of the equation 3𝑥 2 − 5𝑥 − 7 = 0.

Solution and Reflection

𝑎 = 3, 𝑏 = −5, 𝑐 = −7, therefore


−(−5) ± �(−5)2 − 4(3)(−7)
𝑥=
6
5 ± √25 + 84
→ 𝑥=
6
5 ± √109
→ 𝑥=
6
5 + √109 5 − √109
→ either 𝑥 = or 𝑥=
6 6

NB: The conditions of the question say without using a calculator so we stop here. Always
follow the given instruction.

3.4 Quadratic Equations (Revision)


We have covered the laws of the exponents and logarithms in the previous section; we have
begun this section by covering the solving of quadratic equations. We are now in a position to
solve exponential and logarithmic equations. Let’s put our skills to practice by means of
some examples. Remember, all these skills have to be used together.

Learning Activity 3.4.1: Remembering Quadratic Equations

Example 1
Solve the following equations without using a calculator, and simplify your answer.

a) 3𝑒 𝑥 − 3𝑒 2−𝑥 = −8𝑒
b) ln 2𝑥 + ln(3𝑥 − 𝑒) − 2 = ln 8
c)
2 𝑥4
𝑥 ln 𝑒𝑥 =
𝑒

d)
(𝑥 2 − 5)ln 𝑥 = 𝑥

24
Solution and Reflection

a)
3𝑒 𝑥 − 3𝑒 2−𝑥 = −8𝑒
𝑥
𝑒2
→ 3𝑒 − 3 𝑥 = −8𝑒
𝑒
→ 3(𝑒 𝑥 )2 − 3𝑒 2 = −8𝑒 ∙ 𝑒 𝑥
→ 3(𝑒 𝑥 )2 + 8𝑒 ∙ 𝑒 𝑥 − 3𝑒 2 = 0
→ 3(𝑒 𝑥 )2 + 9𝑒 ∙ 𝑒 𝑥 − 𝑒 ∙ 𝑒 𝑥 − 3𝑒 2 = 0
→ 3𝑒 𝑥 (𝑒 𝑥 + 3𝑒) − 𝑒(𝑒 𝑥 + 3𝑒) = 0
→ (3𝑒 𝑥 − 𝑒)(𝑒 𝑥 + 3𝑒) = 0
→ either (3𝑒 𝑥 − 𝑒) = 0 or (𝑒 𝑥 + 3𝑒) = 0
𝑒
→ either 𝑒 𝑥 = or 𝑒 𝑥 = −3𝑒 (not feasible)
3
𝑒
→ 𝑒𝑥 = only
3
𝑒
→ (Taking the ln both sides) ln 𝑒 𝑥 = ln � �
3
→ 𝑥 = ln 𝑒 − ln 3 = 1 − ln 3

NB: Keynote instruction, without using a calculator so you leave this as the final answer. If
you couldn’t understand any of the earlier steps in this example, please revisit the
factorisation of quadratic expressions. You should also go back to the exponential graphs and
note that 𝑒 𝑥 is not defined less than zero. Hence we discard 𝑒 𝑥 = −3𝑒 and put a note that it is
not feasible. You will be penalised if you don’t discard the infeasible solution. Also, do
not use a calculator to get the value ln 3 unless specifically instructed to do so.

b)
ln 2𝑥 + ln(3𝑥 − 𝑒) − 2 = ln 8

To solve these types of questions involving sums of logarithms, the first step you should
perform is ‘collect like terms’ by moving all the logarithmic terms to the left-hand side and
all the other terms to the right-hand side. After that, you simplify the left-hand side using the
laws of logarithms so that you have a single term on both the left and the right-hand side.
From there, you take the antilogarithms and proceed to solve the quadratic equation.
Remember, 𝐥𝐧(𝟑𝒙 − 𝒆) ≠ 𝐥𝐧 𝟑𝒙 − 𝐥𝐧 𝒆 never!
Let’s see how this actually works.

ln 2𝑥 + ln(3𝑥 − 𝑒) − 2 = ln 8
→ ln 2𝑥 + ln(3𝑥 − 𝑒) − ln 8 = 2
→ ln�2𝑥(3𝑥 − 𝑒)� − ln 8 = 2
2𝑥(3𝑥 − 𝑒)
→ ln =2
8
2𝑥(3𝑥 − 𝑒)
→ = 𝑒2
8
→ 6𝑥 2 − 2𝑒𝑥 = 8𝑒 2

25
→ 6𝑥 2 − 2𝑒𝑥 − 8𝑒 2 = 0
→ 3𝑥 2 − 𝑒𝑥 − 4𝑒 2 = 0
→ 3𝑥 2 + 3𝑒𝑥 − 4𝑒𝑥 − 4𝑒 2 = 0
→ 3𝑥(𝑥 + 𝑒) − 4𝑒(𝑥 + 𝑒) = 0
→ (3𝑥 − 4𝑒)(𝑥 + 𝑒) = 0
→ either (3𝑥 − 4𝑒) = 0 or (𝑥 + 𝑒) = 0
4𝑒
→ 𝑥= or 𝑥 = −𝑒
3

So these are our final two solutions, right? Wrong In case you haven’t met this kind of a
question before, you should flip back to the previous section on the graph of a logarithm and
from there you will observe that the ln of a negative number is undefined. Look closely at
the original equation:

ln 2𝑥 + ln(3𝑥 − 𝑒) − 2 = ln 8

Of our two solutions, 𝑥 = −𝑒 is negative hence we discard it since ln(−2𝑒) is undefined,


giving the final answer as:

4𝑒
𝑥= only.
3

NB: You will be penalised if you don’t discard the infeasible solution. Do not use a
𝟒𝒆
calculator to evaluate 𝒙 = 𝟑 , you may be penalised for it.

c)
ln 𝑒𝑥 2
𝑥4
𝑥 =
𝑒
ln 𝑒𝑥 2
𝑥4
→ ln�𝑥 � = ln � �
𝑒
(Remember you take the ln on both sides; this is an equation. Whatever you do on the left
hand side, you also do on the right hand side simultaneously)
→ (ln 𝑒𝑥 2 ) (ln 𝑥) = ln 𝑥 4 − ln 𝑒
→ (ln 𝑒 + ln 𝑥 2 )(ln 𝑥) = 4 ln 𝑥 − 1
→ (1 + 2 ln 𝑥)(ln 𝑥) = 4 ln 𝑥 − 1
→ ln 𝑥 + 2(ln 𝑥)2 = 4 ln 𝑥 − 1
→ 2(ln 𝑥)2 − 3ln 𝑥 + 1 = 0
→ 2(ln 𝑥)2 − 2 ln 𝑥 − ln 𝑥 + 1 = 0
→ 2 ln 𝑥(ln 𝑥 − 1) − (ln 𝑥 − 1) = 0
→ (2 ln 𝑥 − 1)(ln 𝑥 − 1) = 0
→ Either (2 ln 𝑥 − 1) = 0 𝑜𝑟 (ln 𝑥 − 1) = 0
1
→ ln 𝑥 = or ln 𝑥 = 1
2
1
→ 𝑥 = 𝑒 2 or 𝑥 = 𝑒

In this case, there is no cause for contradiction hence both solutions are feasible. Do not use
a calculator to evaluate either of the two solutions unless specifically instructed
otherwise.

26
d)

(𝑥 2 − 5)ln 𝑥 = 𝑥
→ ln(𝑥 2 − 5)ln 𝑥 = ln 𝑥
→ ln 𝑥 ∙ ln(𝑥 2 − 5) = ln 𝑥

Don’t make the mistake of dividing throughout by ln x as this means you lose a solution
→ ln 𝑥 ∙ ln(𝑥 2 − 5) − ln 𝑥 = 0
→ ln 𝑥 ∙ (ln(𝑥 2 − 5) − 1) = 0
→ Either ln 𝑥 = 0 or ln(𝑥 2 − 5) − 1 = 0
→ 𝑥 = 𝑒 0 or (𝑥 2 − 5) = 𝑒 1
→ 𝑥 = 1 or 𝑥 2 = 𝑒 + 5
→ 𝑥 = 1 or 𝑥 = √𝑒 + 5

I have discarded the second part of the second solution, 𝑥 = −√𝑒 + 5. Can you tell why? Do
not use a calculator to evaluate 𝒙 = √𝒆 + 𝟓, you may be penalised for it.

3.5 Self Study


Clearly, the laws of exponents and logarithms work together in solving an exponential or a
logarithmic equation. Now it’s your chance to apply this knowledge to test yourself how far
you are in this area.

Exercise 1
Turn to page 22 of the Maths 1 study guide, and answer all of Question 5. The instruction
says, ‘Solve for 𝑥 in the following equations without using a calculator, and simplify your
answer’. Please abide by this instruction.

REFLECTION
In this topic on Exponential and Logarithmic equations, we have reviewed the laws of
exponents and logarithms that we covered in the previous section. To solve a logarithmic or
exponential equation, we apply these laws step by step until we get our known. To solve an
exponential equation, start by clearing all fractions by multiplying throughout by the common
denominator and then clear all negative indices by multiplying by the inverse. Use of a
calculator is strongly discouraged unless a question says otherwise. More often than not,
when solving an exponential or logarithmic equation, we end up solving a quadratic equation.
Quadratic equations can be solved by direct factorisation, factorisation by grouping, or
completing the square which is commonly known as the quadratic formula. After getting the
solution/s, you need to test if they are feasible solutions by substituting back into the
logarithmic or exponential equation. Always discard the unwanted solution as failure to do so
will certainly lead to loss of marks. When the equation requires the use of a calculator,
always round off the solution to the stated degree of accuracy.

We now move on to applications of exponential and logarithmic equations in our next section
on ‘Exponential Growth and Decay’.

27
28
4 Applications: Exponential Growth and Decay
4.1 Learning Outcomes
After completing this section, you should be able to
• Apply the properties of the natural logarithm and base e to a function of the form
y = Aekx to solve either a natural growth problem ( k > 0) or a natural decay problem
( k < 0)

4.2 Introduction
There are many situations in life where a quantity grows or decays by a constant factor over
equal time intervals. Exponential functions are important because they provide mathematical
models for situations like these. To be able to utilise these functions, you need to be ‘fluent’
in the solving of exponential and logarithmic equations and therefore, the laws of exponents
and logarithms.

Learning Activity 4.2.1: Solving Exponential Growth and Decay Equations

We start this section by reviewing some of the concepts that you have covered so far from the
previous sections. I will demonstrate some examples to help you refresh and leave you work
out the rest.

Example
Solve for x in the following equations without using a calculator
1. 3𝑥 = 5
2. 2𝑥−1 = 7
3. 52𝑥+1 = 32−𝑥
4. 0.6𝑥 = 0.4
5. 0.8𝑥−1 = 0.2𝑥
6. 10 = 50(0.75)𝑥

Solution and Reflection

1)
3𝑥 = 5
→ ln 3𝑥 = ln 5
→ 𝑥 ln 3 = ln 5
ln 5
→ 𝑥=
ln 3

NB: If you are told to give the answer rounded off to a certain number of decimal places
or significant figures, you leave the calculation at this stage. There should be no use of a
calculator in any of the earlier steps

29
2)
2𝑥−1 = 7
→ ln 2𝑥−1 = ln 7
→ (𝑥 − 1) ln 2 = ln 7
ln 7
→ 𝑥−1=
ln 2
ln 7
→ 𝑥 = 1+
ln 2
NB: The comment in the previous question applies.

3)
52𝑥+1 = 32−𝑥
→ ln 52𝑥+1 = ln 32−𝑥
→ (2𝑥 + 1) ln 5 = (2 − 𝑥) ln 3
→ 2𝑥 ln 5 + ln 5 = 2 ln 3 − 𝑥 ln 3
→ 2𝑥 ln 5 + 𝑥 ln 3 = 2 ln 3 − ln 5
→ 𝑥(2 ln 5 + ln 3) = ln 9 − ln 5
9
ln
→ 𝑥= 5
ln 25 + ln 3
9
ln
→ 𝑥= 5
ln 75

NB: See question 1 for comment on the use of a calculator. Make sure you are able to
follow through the working in this question and identify the laws used.
There is one common mistake that students in this course typically make and I need it to
address it.

Mistake 5
𝐥𝐧 𝒙
≠ 𝐥𝐧 𝒙 − 𝐥𝐧 𝒚
𝐥𝐧 𝒚
Never!
The source of the confusion is in applying the law of logarithms given below:

𝑥
ln = ln 𝑥 − ln 𝑦
𝑦
𝐥𝐧 𝒙 𝐥𝐧 𝒙
=
𝐥𝐧 𝒚 𝐥𝐧 𝒚
And that’s the end of the story; you cannot simplify the above expression any further if the
terms are already simplified.

4)
0.6𝑥 = 0.4
→ ln 0.6𝑥 = ln 0.4
→ 𝑥 ln 0.6 = ln 0.4
ln 0.6
→ 𝑥=
ln 0.4

30
NB: No further meaningful simplification is possible.

5)
0.8𝑥−1 = 0.2𝑥
→ ln 0.8𝑥−1 = ln 0.2𝑥
→ (𝑥 − 1) ln 0.8 = 𝑥 ln 0.2
→ 𝑥 ln 0.8 − ln 0.8 = 𝑥 ln 0.2
→ 𝑥 ln 0.8 − 𝑥 ln 0.2 = ln 0.8
→ 𝑥(ln 0.8 − ln 0.2) = ln 0.8
ln 0.8
→ 𝑥=
0.8
ln 0.2
ln 0.8
→ 𝑥=
ln 4
NB: Again, no further meaningful simplification is possible.

6)
10 = 50(0.75)𝑥
10
→ = 0.75𝑥
50
1
→ ln = ln 0.75𝑥
5
→ ln 5−1 = 𝑥 ln 0.75
ln 5
→ − =𝑥
ln 0.75
ln 5
→ 𝑥=−
ln 0.75

NB: Don’t forget the negative sign; your solution is incorrect without it. Read back on
the relevant law to see where it comes from.

Learning Activity 4.2.2: Growth and Decay examples

Now that you are armed with the necessary arithmetical skills, it is time to reflect on this
section by means of some relevant examples. Problems in this domain typically come in the
form of word problems, so you need to read and pay particular attention to detail. You should
focus your concern on the units used in the formula, and the units required in the solution;
they may not be the same. You will be penalised if you give a solution in units other than
those requested in the question even if the two are of equivalent value.

Question 1
3𝑡

A radioactive substance decays according to the formula 𝑆 = 𝑆0 𝑒 50 with t the time in
years and 𝑆0 the original amount of the substance in grams.
a) If 𝑆0 was 50g, how much will be left after 12 years?
b) How long does it take until there are 20g left?

31
Solution and Reflection
3𝑡

Since 𝑆0 = 50, the formula therefore becomes 𝑆 = 50𝑒 50 . Carefully note that S represents
the amount of radioactive substance left after time t, pay special attention to whether the
question refers to how much material is left or how much material has decayed. The
question can come either way and you are expected to discern the difference.
3𝑡
a) So in this case, 𝑆 = 50𝑒 −50 and 𝑡 = 12, therefore
3×12
𝑆 = 50𝑒 − 50 = 24.3𝑔
correct to 1 decimal place.
3𝑡
b) We again take the formula 𝑆 = 50𝑒 −50 with 𝑆 = 20; substitute and get
3𝑡
20 = 50𝑒 −50
20 3𝑡
→ = 𝑒 −50
50
2 3𝑡
→ ln = −
5 50
50 2
→ − ln = 𝑡
3 5
50 2
→ 𝑡 = − ln
3 5
→ 𝑡 = 15.3 years

I hope you have been following the exercises we did at the beginning of this section. The
calculator should only be introduced at the very last step, there should be no
‘intermediate round-offs’ in your build up to the answer. Another thing, don’t forget the
negative sign; your solution cannot be correct without it. A solution like t = -15.3 years
should be enough to raise a red flag that you may have left a negative sign somewhere.
In case the above question stimulated your interest but you still want additional practice,
here’s another question for you.

Question 2
A radioactive substance is decaying according to the formula 𝑦 = 𝐴𝑒 𝑘𝑡 , where t is time in
years. The initial amount 𝐴 = 10𝑔 and 8g remains after 5 years.
a) Find k. Leave your answer in terms of natural logarithms.
b) Estimate the amount remaining after 10 years.
c) How long does it take for 4�5 of the radioactive substance to decay?
d) Find the half-life to the nearest tenth of a year.

Solution and Reflection

a) 𝐴 = 10, therefore the formula becomes 𝑦 = 10𝑒 𝑘𝑡 . Also, we are told that 8g remains
after 5 years therefore, we substitute these two values into the formula to get

8 = 10𝑒 5𝑘

32
8
→ = 𝑒 5𝑘
10
4
→ ln = 5𝑘
5
1 4
→ ln = 𝑘
5 5
1 4
→ 𝑘 = ln
5 5

No further meaningful simplification is possible as we are told to leave the answer in terms of
natural logarithms.
1 4
b) With the answer in (a), the formula is now 𝑦 = 10𝑒 5 ln5𝑡 . To get the amount
remaining after 10 years, we simply substitute 𝑡 = 10 into the equation to get
1 4 1 4
𝑦 = 10𝑒 5 ln5𝑡 = 10𝑒 �(5 ln5)×10� = 6.4𝑔

This is correct to 1 decimal place. Where the degree of accuracy is not specified, it is
sufficient to give the solution correct to 1 decimal place.

c) This question will sink the gullible, if you don’t read closely; you too will fall for it.
The question is asking for the time taken for 4�5 of the substance to decay, not the
time taken for the substance to reduce to 4�5 of its original value. So, we have to turn
it around, we note that for 4�5 of the material to decay, it will mean that 1�5 of the
material remains. You should never be so pressed for time that you can’t read the
finer details, you lose everything and all your effort will come to nought needlessly.
Now let’s answer the question:

When 4�5 of the substance has decayed, it means 1�5 of the substance remains, implying that
1
𝑦 = 5 × 10 = 2𝑔. Substituting for 𝑦 = 2 in the formula in (b), we get

1 4
2 = 10𝑒 (5 ln5)𝑡
2 1 4
→ = 𝑒 �5 ln5�𝑡
10
1 1 4
→ ln = � ln � 𝑡
5 5 5
ln 5
− =𝑡
1 4
� ln �
5 5
36.1 = 𝑡
𝑡 = 36.1 years

Again, observe how I introduce the calculator at the very last step, avoid intermediate
round-offs. Also observe how introducing braces (brackets) in the first step helps make your
work look more presentable and protects you from making unintended mistakes. The reason
why I gave the answer to 1 decimal place should be now apparent by now, otherwise read the

33
previous question. Units are extremely important and should never be forgotten in your
final answer.

d) Before I give the solution, I need to give the definition of a half-life. Some students do
get the final answer correct on the half-life but only by accident. When a substance is
decaying naturally, the time taken for one-half of the original quantity to decay
is called the half-life.

To answer this question, we begin by saying, when half of the original quantity has decayed,
the amount remaining or y is given by

1 1
𝑦= × 𝐴 = × 10 = 5
2 2

Therefore, substituting for 𝑦 = 5 into the formula in (b), we have


1 4
5 = 10𝑒 �5 ln5�𝑡
5 1 4
→ = 𝑒 �5 ln5�𝑡
10
1 1 4
→ ln = � ln � 𝑡
2 5 5
ln 2
− =𝑡
1 4
� ln �
5 5
ln 2
→ 𝑡=− = 15.5 years
1 4
� ln �
5 5

Rounding off
Students sometimes get confused in rounding off and end up giving all sorts of answers.
What does it mean to say round of to 1 decimal place, 2 decimal places, 3 significant figures,
nearest tenth, etc.? We will address this by means of an illustrative example.

Example
Round off the number 75.0794 to the nearest:
a) 1 decimal place
b) 2 d.p.
c) 3 d.p.
d) 1 significant figure
e) 2 s.f.
f) 3 s.f.
g) tenth
h) hundredth
i) ten
j) hundred

Solution
a) 75.0794 = 75.1 correct to 1 d.p. (7 is greater than 5)
b) 75.0794 = 75.08 correct to 2 d.p.

34
c) 75.0794 = 75.079 correct to 3 d.p.
d) 75.0794 = 80 correct to 1 s.f.
e) 75.0794 = 75 correct to 2 s.f.
f) 75.0794 = 75.1 correct to 3 s.f.
g) 75.0794 = 75.1 correct to the nearest tenth
h) 75.0794 = 75.08 correct to the nearest hundredth
i) 75.0794 = 80 to the nearest ten
j) 075.0794 = 100 to the nearest hundred

NB: The underlined figure shows the figure you have to round off. If the figure next to that
figure is 5 or more, you add 1 otherwise you leave in unchanged.
To round off to the nearest tenth means the same as to round off to the nearest 1 decimal
place. Likewise to round off to the nearest hundredth means the same as to round off to the
nearest 2 decimal places; etc. Rounding off to the nearest tenth does not mean you
𝟏
multiply the answer by 𝟏𝟎 or round it off to the nearest ten as I have observed when
marking previous papers.

If you are asked to round off to the nearest 1 decimal place and you give your solution
as say 75.10, you will be penalised. Numbers rounded off to the nearest 1 decimal place
should have only one decimal place and numbers rounded off to the nearest 2 decimal places
should be written with only 2 decimal places.

4.3 Self Study


Having covered all the elements of the learning outcomes for this section, it is now time for
you to assess yourself. As usual, you should take your solutions to the next tutorial session
for the tutor to assess your efforts.

Complete Question 6 in the Maths 1 study guide, pages 22-23.

REFLECTION
In this section of the course, we were extending the application of the laws of logarithms and
exponents to solve problems involving exponential growth and decay. The general equation
for exponential growth or exponential decay is
y = Ae kx
In the case of exponential growth, k > 0; and
in the case of exponential decay, k < 0.
By taking the natural logarithm on the equation for natural growth/decay, we can easily solve
for either k or x.
Always stick to the required degree of accuracy.

35
5 Radian Measure
Learning Outcomes
After completing this section, you should be able to
• Do the conversion of measures of angles between degrees and radians
• Use radian measure in calculating arc length, radius, area of a sector and area of a
segment

5.1 Introduction
1
The practical unit of measurement for angles is the degree (°) which is 360th of a complete
revolution. The number 360 comes from Babylonian times but it is an arbitrary choice. There
is another system of measurement called circular or radian measure which is more suitable
for advanced mathematics like Engineering Mathematics.

5.2 Radian Measure


𝝅 𝐫𝐚𝐝 = 𝟏𝟖𝟎°
Therefore,
𝜋 𝜋
rad = 90°; rad = 45°; etc
2 4
Simple proportion using cross-multiplication will yield the desired conversion.

It is time to refresh your memory on the conversion. When converting from degrees
to radians, always leave your answer in terms of π.

Question
Convert the following radians to degree measure:
𝜋
a) 3
2𝜋
b) 3
11𝜋
c) 6
5𝜋
d) 4

Question
Convert the following to radian measure as a multiple of π.
a) 30°
b) 135°
c) 105°
d) 40°
e) 200°
f) 400°
g) 75°

Always remember to express your answers in lowest terms.

36
Length of an arc
Consider an arc of length s in a circle with radius r. The arc subtends an angle of θ radians at
the centre. The formula to evaluate the length of the arc is given by
𝑠 = 𝑟𝜃
This formula is only valid if θ is in radians.

Example
Find the length of an arc which subtends an angle of 135° at the centre in a circle of radius 8
cm.

Solution
First convert the angle to radians. Thus we have
135 3
135° = × 𝜋 = 𝜋 radians
180 4
From the formula 𝑠 = 𝑟𝜃 (which will be supplied in the list of formulas in any assessment),
we have
3𝜋
𝑠 = 𝑟𝜃 = 8 × = 6𝜋 cm
4
If the question had requested the solution to a certain degree of accuracy, you could then use
the calculator at this stage.
Area of a Sector of a circle
The diagram below shows a sector AOB of angle 𝜃 in a radius with centre O and radius r.
The area of the sector is given by
1
Area of sector of triangle of angle θ = 𝑟 2 𝜃
2

Once again, this formula is only valid in radians.

Example
If the area of a sector in a circle is 9 cm2 and the central angle of the sector is 120°, find the
radius of the circle without using a calculator.
𝑟2𝜃
�𝑈𝑠𝑒: 𝐴𝑟𝑒𝑎 = 2
�.

Solution
The formulas to use in radian measure are supplied either within the question itself or in the
formula sheet at the back. Now back to the question.

37
The first step is to convert the given angle to radians. That is

120 2𝜋
120° = ×𝜋 =
180 3

Since we want to find the radius, the next step is to re-arrange the above formula so that we
make r the subject of the formula.
𝑟 2𝜃
𝐴=
2
→ 2𝐴 = 𝑟 2 𝜃
2𝐴
→ = 𝑟2
𝜃
2𝐴
→ 𝑟=�
𝜃
2𝜋
Now, substitute 𝐴 = 9 and 𝜃 = 3
into the above formula, we get

2×9 18 3 27
𝑟=� =� = �18 × = � cm
2𝜋 2𝜋 2𝜋 𝜋
�3� �3�

The instruction is very clear in this case, without using a calculator. The above therefore
serves as your final answer.
Area of a triangle

Consider the shape above, which is our sector. To get the area of the triangle OAB, we use
the formula:
1 1
Area of triangle AOB = 𝑏𝑐 sin 𝐴 = 𝑟 2 sin 𝜃
2 2
Area of a segment
The segment is the region between the arc AB and the line AB in the diagram. We get the
area of the segment by getting the difference between the area of the sector OAB and the area
of the triangle OAB. The formula for the area of a segment is thus

Area of the segment = Area of sector OAB − Area of triangle OAB

38
1 1
= 𝑟 2 𝜃 − 𝑟 2 sin 𝜃
2 2
1 2
= 𝑟 (𝜃 − sin 𝜃)
2

NB: This formula is only valid in radians, and you have to convert the angle to radians first
before proceeding with your calculations.

Example
1
The area of a segment of a circle is given by the formula 𝐴 = 2 𝑟 2 (𝜃 − sin 𝜃). Find A to three
decimal places if 𝑟 = 6cm and 𝜃 = 60°.

Solution
You first convert the angle to radians, that is
60 𝜋
60° = × 𝜋 = radians
𝜋
180 3
Now, substitute for 𝑟 = 6cm and 𝜃 = 3 into the given formula to get
1
𝐴 = 𝑟 2 (𝜃 − sin 𝜃)
2
1 𝜋 𝜋
= × 62 × � − sin �
2 3 3
= 3.261 cm2 to 3 d. p.

This step needs to be shown even if the actual calculation is going to be done using a
calculator. Change your calculator to radian mode before you start punching in the
above formula.

Keynote: When you are using a calculator to evaluate a formula, you should substitute the
variable into the formula and enter it into the calculator without going through any
intermediate stages. There’s no need to show any other working here after showing that the
variables have been substituted

For your learning activity, please turn to page 23 of the guide and work out
Question 1, Tutorial 2.

REFLECTION
In this section, we looked at the concept of the radian measure. The radian is an alternative
unit used to measure an angle. We showed how to evaluate the length of an arc, the area of a
sector and the area of a triangle subtended by the segment of a circle using the concept of a
radian measure. The main thing to remember is that all the formulae given in this section are
only valid if the angle is measured in radians. Therefore, always change the angle to radians,
preferably in multiples of π before applying the formulae. When doing the computations,
always change the calculator to radian mode before you start any calculations.

39
6 The Binomial Theorem
Learning Outcomes
After completing this section, you should be able to
• Write down the Binomial Expansion for all rational powers of n

The Binomial expansion for ( a + b )


n
is given by the following formula
b2 b3
(a + b) =a n + na n −1b + n ( n − 1) a n − 2 + n ( n − 1)( n − 2 ) a n −3 +  + b n
n

2! 3!

where n is an integer and n ! is called the factorial of n or alternatively n factorial and is


evaluated by
n ! =1× 2 × × ( n − 1) × n.

Example

= ( 4 )( 3)( 2 )(1) 120


5! 5=
= ( 3)( 2 )(1) 24
4! 4=
etc
You can easily evaluate n ! from the calculator.

Example
Use the Binomial Theorem to evaluate ( a + b ) .
4

Solution

Since n = 4, we have
b2 b3
+ 4 ( 3)( 2 ) a + b 4
( a + b ) =a 4 + 4a3b + 4 ( 3) a 2
4

2! 3!
You can easily verify that 2! = 2 and 3! = 6 therefore giving you after simplification:
( a + b ) =a 4 + 4a3b + 6a 2b 2 + 4ab3 + b 4
4

Exercise
Use the Binomial Theorem to evaluate the following:
(a + b)
3
a)
(a + b)
5
b)

Pascal’s triangle
A more convenient way of evaluating the coefficients of the terms in the Binomial Expansion
is the Pascal’s triangle. The coefficients of the Binomial Expansion for n = 1 to n = 3 are
shown below:

40
n =1 1 1
n=2 1 2 1
n=3 1 3 3 1
Breaking it down, we have
( a + b ) =a + b
1

( a + b ) =a 2 + 2ab + b 2
2

( a + b ) =a3 + 3a 2b + 3ab2 + b3
3

Note that the first and last coefficients of the Pascal’s triangle in each row are 1. We obtain
the other coefficients by adding the two nearest coefficients to get the next coefficient of the
next row, for example, 1 + 2 = 3; 2 + 1 =3; etc. Successive rows are obtained this way.

Exercise
a) Continue the Pascal’s triangle till you reach n = 7.
Use the Binomial Theorem and Pascal’s triangle to expand ( a + b ) .
4
b)

Example
Use the Binomial’s Theorem to expand
( 3x + 2 )
3
a)
( 3x − 2 )
3
b)

Solution

a) a = 3 x, b = 2, and n = 3. We use Pascal’s triangle to obtain the coefficients of the


expansion. Therefore, we have
( a + b ) =a3 + 3a 2b + 3ab2 + b3
3

( 3x + 2 ) = ( 3x ) + 3 ( 3x ) ( 2 ) + 3 ( 3x )( 2 ) + ( 2 )
3 3 2 2 3

= 27 x 3 + 54 x 2 + 36 x + 8
b) Similar to (a) above, except for b = −2. It therefore follows that
( 3x − 2=
) ( 3x ) + 3 ( 3x ) ( −2 ) + 3 ( 3x )( −2 ) + ( −2 )
3 3 2 2 3

You need to be careful with the treatment of the negative sign. Simplifying, we have
( 3x − 2 ) = 27 x3 − 54 x 2 + 36 x − 8
3

Exercise
Expand and simplify the given expressions by the use of Binomial’s Theorem and Pascal’s
triangle.
( 5 x − 3)
4
a)
( x − 4)
5
b)
( 2a + 1)
6
c)
( x − 3)
7
d)

41
Binomial Theorem when n is rational or negative
Suppose we have the expression ( a + b ) . How can we apply the Binomial Theorem to
−1

expand this expression?


One thing you will immediately notice is that Pascal’s triangle is not applicable as n is not an
integer. Furthermore, the number of terms in the Binomial expansion of ( a + b ) is infinite
−1

unlike in the case when n is an integer as we saw earlier.

To expand ( a + b ) where n is a negative or rational number (fraction); we use the general


n

Binomial Theorem:
2 3
n−2 b n −3 b b
( a + b ) =a + na b + n ( n − 1) a
n −1
+ n ( n − 1)( n − 2 ) a + ; <1
n n

2! 3! a
Take note that since this is an infinite series, you will be only expected to show only the first
few terms, normally 4 but this depends on the exam.
b
The condition < 1 is meant to ensure that the infinite series converges and should be
a
included always when dealing with an infinite series.

Exercise
Find the first four terms in the Binomial expansion of the following expressions.
( x + 3)
1
a) 2

(3 + x )
1
b) 2

Solution
a) In this case, a = x, b = 3, and n = 12 . Therefore, from the Binomial Theorem, we have
b2 b3
(a + b) =a + na b + n ( n − 1) a
n −1
+ n ( n − 1)( n − 2 ) + 
n−2
n n

2! 3!
 1  −1  1  1  −3  3   1  1  3  −5  3 
2 3
( x + 3) x 2 +   x 2 ( 3) +    −  x 2   +    −   −  x 2   + 
1
=
1
2

2  2  2   2!   2   2   2   3! 
9 3
27
=x+ + 2 −
+ ; x >3
2 x 8x x 8x x
b) This is very similar to (a) except that the terms inside the brackets have been
interchanged. We now have a = 3, b = x, and n = 12 . Hence we have
b2 b3
( a + b ) =a + na b + n ( n − 1) a
n −1
+ n ( n − 1)( n − 2 ) + 
n−2
n n

2! 3!
 1  − 12
( ) ( ) ( )
 1   1  − 32  x   1   1   3  − 52  x 
2 3
( ) ( )   
1
+ =
1
3 x 2
3 2
+   3 x + − 3 +
    −  −  3   +
2  2  2   2!   2   2   2   3! 
2 3
x x x
=3+ − + + ; x <3
2 3 24 3 144 3
As you can see, the (a) and (b) differ in that in (a), we have the Binomial Expansion in
terms of increasing powers of 3 or equivalently, in terms of x ; whereas in (b), we
have the Binomial Expansion in terms of increasing powers of x or equivalently, in

42
terms of 3. Also notice that the difference in ordering of the terms leads to different
b
conditions for < 1 thereby resulting in different conditions after simplification.
a

Exercise
Use the Binomial expansion to find the first four terms in the expansion of
a) (1 + x )
−1

(1 + x )
− 12
b)
(1 + 2x )
−1
c)

(1 − 3x )
− 1
d) 2

1
e)
2+ x
x+3
f)
x2 − 9
g)
Solution
The first 4 easily follow from what we have done so far and are left as exercises.
1
to the indical form ( 2 + x ) 2 . In terms
−1
e) We first need to covert the expression
2+ x
1
of the Binomial Expansion, we have a = 2, b = x, and n = − . Therefore
2

b2 b3
(a + b) =a n + na n −1b + n ( n − 1) a n − 2
+ n ( n − 1)( n − 2 ) a n −3 + 
n

2! 3!
 1  − 32  1   3  − 52  x   1   3   5  − 72  x 
2 3
(2 + x) = 2 +  −  2 ( x) +  −   −  2   +  −   −   −  2   +
− 12 − 12

 2  2  2   2!   2   2   2   3! 
1 x 3x 2 5 x3
= − + − + ; x <2
2 4 2 32 2 128 2

x+3 x+3 1
= ( x − 3)
−1
f) We start by simplifying the expression as = =
x − 9 ( x + 3)( x − 3) x − 3
2

where x + 3 ≠ 0. Expansion by the Binomial Expansion immediately follows subject


3
to the conditions x ≠ −3 and < 1 ⇒ x > 3.
x

Example
1
Use the Binomial Theorem to find the first four terms in the expansion of .
(3 − ln e 2 x ) 2

43
Solution
Questions you face in this course typically require the applications of more than one concept.
In this case for example, you should first apply the laws of logarithms before performing the
Binomial Expansion. Therefore, from the laws of logarithms ln e x = x, we say
1 1
= (3 − 2x )
−2
=
( 3 − ln e2 x ) ( 3 − 2 x )
2 2

Hence we now have a = 3, b = −2 x, and n = −2. The Binomial Expansion immediately


follows:
b2 b3 b
( a + b ) =a n + na n−1b + n ( n − 1) a n−2 + n ( n − 1)( n − 2 ) + ; <1
n

2! 3! a
You should be able to take it from here.

Attempt the following questions for your tutorial.

Question
Use the Binomial Expansion to find the first four terms in the expansion of the following
terms
a)
1
(1 + 𝑥)2

b)
1
1 − 2𝑥
c)
𝑥−3
� 2
𝑥 −9
Binomial Series
In the Binomial Expansion of ( a + b ) , when we have a = 1, the Binomial Expansion
n

reduces to a Binomial series in terms of increasing powers of b.


b2 b3
(1 + b ) =1 + nb + n ( n − 1) + n ( n − 1)( n − 2 ) +  , b <1
n

2! 3!

Example:
1
Find the first 4 terms in the Binomial Expansion of .
(1 − 2x )
2

Solution:
1
(1 − 2 x )
−2
=
(1 − 2 x )
2

Therefore, we have a = 1, b = −2 x, and n = −2. By inspection, since a = 1, you can observe


that the expansion gives rise to a Binomial Series with b = −2 x, and n = −2 :

44
b2 b3
(1 + b ) 1 + nb + n ( n − 1)
= + n ( n − 1)( n − 2 ) +  , b <1
n

2! 3!
( −2 x ) ( −2 x )
2 3

(1 − 2 x ) = 1 + ( −2 )( −2 x ) + ( −2 )( −3) + ( −2 )( −3)( −3)


−2
+ , 2x < 1
2! 3!
1
=+
1 4 x + 12 x 2 + 24 x 3 +  , x<
2

REFLECTION
In this section, we covered the series expansion by the Binomial Theorem.
• The formula for the Binomial Theorem when n is an integer
2 3
n−2 b n −3 b
( a + b ) =a + na b + n ( n − 1) a
n −1
+ n ( n − 1)( n − 2 ) a +  + bn
n n

2 3!
• The Binomial Expansion for ( a + b ) when n is negative or rational is the infinite
n

series:
b2 b3 b
( a + b ) =a n + na n−1b + n ( n − 1) a n−2 + n ( n − 1)( n − 2 ) a n−3 +  , <1
n

2! 3! a

• In the special case when a = 1, the Binomial Theorem expansion reduces to the
Binomial series:
b2 b3
(1 + b ) =1 + nb + n ( n − 1) + n ( n − 1)( n − 1) +  , b <1
n

2! 3!

45
CONTENT AREA 2: TRIGONOMETRY

TRIGONOMETRY

Basic Trigonometric Identities Involving all six Trigonometric


Ratios

Graphs of sine functions of the form y = asin bx + c - angles in


radians

Graphs of cosine functions of the form y = acos bx + c - angles


in radians

Inverse trigonometric functions

Solutions of sine and cosine functions

46
7 Trigonometric Graphs
Learning Outcomes
At the end of this section, you should be able to
• Find the amplitude, period, quarter period and phase shift of a given sine or cosine
function
• Sketch the graph of a given sine or cosine function

7.1 The properties of a sine (cosine) wave


The general equation of a sine function is given below:
𝑦 = 𝐴 sin(𝐵𝑥 + 𝐶)

From the above function, we can easily obtain the amplitude, period, quarter period and
phase shift for the sine curve as follows

Amplitude = |𝐴|
2𝜋
Period, T =
𝐵
𝑇
Quarter Period =
4
𝐶
Phase Shift = −
𝐵

Alternatively, you can obtain the phase shift by setting the argument of sin, that is, 𝐵𝑥 + 𝐶 =
0 and solving for x. When you have the above values, you are ready to draw the sine graph. I
am referring to the sine graph in my exposition but everything above applies to the cosine
graph and the procedure for sketching the graph is the same. I will use the following example
to illustrate how you actually sketch a sine or cosine graph.

Example
Find the amplitude, period, quarter period and phase shift of the following function and
sketch the graph:

𝜋 𝜋 2𝜋
𝑓(𝑥) = 2 sin �2𝑥 − � ; 𝑥 ∈ �− ; �
3 3 3

Solution
From the above function, we have
𝜋
𝐴 = 2; 𝐵 = 2; 𝐶=−
3

By definition, Amplitude |A| = |2| = 2.


Taking the modulus of A ensures we take only the positive values since the amplitude is
always positive even if A may be negative. If A is negative, simply ignore the negative
sign.

You will be penalised if you give a negative value for the amplitude.

47
Coming to the period,
2𝜋 2𝜋
𝑇= = =𝜋
𝐵 2 𝜋
The quarter period is simply a quarter of the period and this is equal to 4 .
The phase shift, by definition is given by
𝜋
𝐶 �− 3� 𝜋
Phase shift = − = − =+
𝐵 2 6

Ok, now that we have all this information, what’s next? We look at the range of values of 𝑥
as stipulated in the question, that is
𝜋 2𝜋
𝑥 ∈ �− ; �
3 3

Stick to the given range of values of 𝑥; no more, no less. Remember we have calculated our
𝜋
quarter period as 4 . We use this quarter period to generate what I will call the ‘quarter points’
𝜋
of 𝑥. We generate these quarter points by continuously adding our quarter period 4 starting
with the lower end point and we stop when we reach the upper end point. In this case, let’s
𝜋
set the first point as x0 = − 3 . This implies

𝜋
𝑥0 = − (given)
3
𝜋 𝜋 𝜋
𝑥1 = − + = −
3 4 12
𝜋 𝜋 𝜋
𝑥2 = − + =
12 4 6
𝜋 𝜋 5𝜋
𝑥3 = + =
6 4 12
5𝜋 𝜋 2𝜋
𝑥4 = + = (stop)
12 4 3

So we now have the set of the quarter points of 𝑥 as


𝜋 𝜋 𝜋 5𝜋 2𝜋
𝑥 ∈ �− ; − ; ; ; �
3 12 6 12 3

Having done this, we now obtain the corresponding 𝑦 values by substituting the above
quarter points in turn. It’s best to enter the results into a form of a table as shown below.
Remember to switch your calculator to radian mode first before proceeding with the
calculations.
𝝅
𝒙 𝒚 = 𝟐 𝐬𝐢𝐧 �𝟐𝒙 − �
𝟑
𝜋
− 0
3
𝜋
− -2
12
𝜋
0
6

48
5𝜋
2
12
2𝜋
0
3
To do the calculations fast, you don’t have to re-enter the formula when you compute the
value of the function at the next quarter point. You must make use of the backspace or back
arrow of your calculator and replace the value of 𝑥 with the appropriate one. The next
question is do you really need to calculate all these points, is there not a better way? (I don’t
like using the term short-cut). Let me explore this concept in the discussion below.

The value of ‘quarter points’


Up to now, I hadn’t elaborated on the reason of using the quarter points. If you look again at
the table above, you will notice that the value of 𝑦 at the quarter points is either -2, 0 or 2.
This makes it easy when we want to sketch the function. If you obtain any other values of
the 𝒚 values outside of those 3 then you have definitely made a mistake.

Another item I wish to bring to your attention is the fluctuation of the value of the sine
function. You don’t really need to calculate all the 𝑦 values to complete the above table. You
only need two points, and yes, two points and you can complete even a page of a table for the
sine function. This is because the sine and cosine functions are oscillating functions, they
move from the amplitude, to zero, to –amplitude, to zero, to amplitude, to zero, to –
amplitude, to zero, etc.

I hope you get the picture here. If you get any two points, say the first is the amplitude and
the next is zero, then clearly the next is –amplitude, then zero and it goes on like that.
Sketching the sine function
The final stage is now sketching the sine function which should be very easy given the above
but there are some common mistakes that I observe during marking that I need to address.
First and foremost, the graph you obtain from the above coordinates should look like the one
below:

49
I have not said anything about the phase shift until now. The phase shift by definition is a
shift of the sine/cosine graph along the 𝑥-axis. Remember we calculated our phase shift to be
𝜋
+ 6 . From the graph, we can see that the phase shift represents a shift of the sine curve in the
positive 𝑥 direction; the converse is true for a negative phase shift. If you have done your
calculations for the sketch correctly, the phase shift on the graph should be the same as the
phase shift you calculated at the very first step.

Now let’s look at the common student mistakes in this section of the course.

Mistake 6
The points on the 𝑥- and 𝑦- axis should only be points from the table. Any other points not
included, leave them out. Only the critical points should be shown on the graph. Your first
step in sketching this function is labelling the 5 ‘quarter period’ points on the 𝑥-axis which
are equally spaced. Only after labelling the 𝑥-axis do you estimate where the zero value
should be. There is no space for 1, 2, 3, or any other cardinal points from the Cartesian plane.
The question says sketch not plot, there’s a difference.

Mistake 7
Trying to calculate the value of 𝑦 when 𝑥 = 0. Unless 𝑥 = 0 is itself one of the quarter
points, don’t worry about the value of the function at this point. The question says sketch
not plot, there’s a difference.

Mistake 8
Extending the range of 𝑥 values to values outside the required domain. Follow the given
instructions; it is one of your responsibilities as a student.

Mistake 9
Using freehand to draw the axis of the graph because the question says sketch. Haibo! To say
sketch a graph does not imply that you should be reckless, it simply means you indicate only
the critical points. You will get a zero if you ever attempt drawing the 𝒙- and 𝒚-axis using
freehand.

Mistake 10
Using a ruler to connect the coordinates of the sine/cosine curve. You will be penalised if you
ever attempt connecting the points using a ruler, use freehand. By now you should familiarise
yourself with the shape of the sine/cosine and one of its most defined properties is that it’s
not zig-zag. When you draw using freehand, the hand may shake a little, we understand.
Make sure your hand is in the inside of the curve to get a smooth curve.

Mistake 11
Making a very tiny sketch. For the 𝑥-axis, make sure your quarter points cover at least half
the width of the page remembering to keep them equally spaced by using a ruler. For the 𝑦-
axis, use the lines of the page to mark the points. Your upper value of the amplitude should
be 4 lines above the 𝑥-axis and your lower value of the amplitude should be 4 lines below the
𝑥-axis. Mark only the end points 4 lines up and 4 lines down, no other points should be
marked at this stage. You will appreciate the benefits of this when we move to the next
section when we use the graphs to solve trigonometric equations.

50
Turn to page 24 of your guide and complete all the sub-questions in Question 2. In
the next section, we will see how we get to use these graphs to solve trigonometric equations,
which is just an extension of what we have just done.

REFLECTION
In this section, we looked at how to plot the curve of a sine/cosine graph. The general
equation of a sine (cosine) curve is as follows:
= y A sin ( Bx + C )
where
The amplitude of the curve = |A|,

Period =
B
C
Phase shift = − .
B
The value of a sine (cosine) wave at a ‘quarter period point’ is either 1, 0 or -1. Once you
have come up with the set of coordinates of the quarter period points, you should join these
using free hand.

In the next section, you will see how we can use the trigonometric graphs to solve
trigonometric equations.

51
8 Trigonometric Graphs and Equations
Learning Outcomes
After completing this section, you should be able to:
• Find particular solutions of trigonometric equations using graphs
• Find the general solutions of sine and cosine equations of the form:
a sin(bx + c) + d = 0 or a cos(bx + c) + d = 0.

8.1 Solving trigonometric equation graphically


π
In the last section, we managed to plot the graph of the function y = 2 sin �2x − 3 � and the
graph is hereby reproduced for completeness sake.

Example
a) Without solving the equation, use your graph above that you have just plotted to
determine the value/s of 𝑥 for which
𝜋 𝜋 2𝜋
sin �2𝑥 − � = −1 𝑥 ∈ �− ; �
3 3 3

b) Indicate on the graph by means of the letters A and B where you would read off the
solution to
𝜋
7 sin �2𝑥 − � = 3.5
3

Solution
The question clearly states ‘without solving the equation’; follow the given instruction. You
will get a zero if you obtain your solution by solving the equation, even if the answer is
the correct one.
52
a) The function is not exactly the same as the one in the graph but the argument of sine
is the same. Therefore, we should be able to manipulate the function without violating
the instruction to get our solution. When we say without solving the equation it
does not mean that you cannot perform some calculations. On the contrary, you
won’t succeed if you don’t do some calculations. To solve this question, we
perform the following calculations
𝜋
sin �2𝑥 − � = −1
3
𝜋
→ 2 sin �2𝑥 − � = −2
3
→ 𝑓(𝑥) = −2

All the above steps are perfectly legitimate. Next, you go to the graph and draw on the graph
where 𝑦 = −2 and label the value of x where this happens as shown. As you can see from the
𝜋
graph, this value is 𝑥 = − 12. This is indicated in the blue dashed line.
Remember, when you solve, you are looking for the value of 𝑥. You should not forget to
𝜋
actually indicate that 𝑥 = − 12; it’s not enough just to indicate it on the graph.

b) As in the previous question, we start by performing some minor calculations:


𝜋
7 sin �2𝑥 − � = 3.5
3
𝜋
→ sin �2𝑥 − � = 0.5
3
𝜋
→ 2 sin �2𝑥 − � = 1
3
→ 𝑓(𝑥) = 1

So we draw the line 𝑦 = 1 on the graph and mark the 𝑥 values where this line intersects with
the graph as shown below.
What are the values of A and B? Don’t waste your time, it’s not part of the question, all you
are required to do is indicate on the graph not work out the explicit values. Working out the
values is a sheer waste of time as you will not be credited for it.

53
Mistake 12
Labelling the points A and B on the curve and not on the x-axis. Remember, A and B are 𝑥
values and should be indicated on the 𝑥-axis. You will be penalised for incorrect labelling of
the points A and B.

Turn to page 24 of the study guide and do all the sub-question on Question 3.

We now proceed to the next section where we get to solve trigonometric equations using
algebraic means, that is, the general solution.

8.2 General Solution of Trigonometric Equations


If OP is drawn on 𝑥 and 𝑦-axis as shown below and if, for all values of θ, the length of OP
is 𝑟 and the coordinates of P are (𝑥; 𝑦), then the sine, cosine and tangent functions are
defined as follows.
𝑦
sin 𝜃 =
𝑟
𝑥
cos 𝜃 =
𝑟
𝑦
tan 𝜃 =
𝑥

If we make a complete revolution by going anticlockwise about the 𝑥-axis, the values of the
sine, cosine and tangent functions change as we move from one quadrant to the next due to
changes in the sign of 𝑥 and 𝑦. This gives us the high school STC quadrant diagram as
shown below.

54
What the above diagram means is that all the three functions are positive in the 1st quadrant,
sine is positive in the 2nd quadrant, tangent is positive in the 3rd quadrant and cosine is
positive in the 4th quadrant. Every function is positive in the 1st quadrant and one other. Also,
every function is negative in 2 quadrants. Therefore, you will always have 2 solutions for
each function in the range [−𝜋; 𝜋].
To solve a basic equation, such as sin 𝜃 = 𝑘,
Step 1: find the 1st quadrant angle α for which sin 𝛼 = |𝑘|;
Step 2: find the quadrants on which θ lies;
Step 3: Determine the corresponding angle for those quadrants

Example
Solve sin 𝜃 = −0.5 for −𝜋 ≤ 𝜃 ≤ 𝜋

Solution
𝜋
Let sin 𝛼 = 0.5, then 𝛼 = 6 .
θ will lie in the 3rd and 4th quadrants ( 𝜃 − 𝜋 and − 𝜃)
𝜋 𝜋
Then, 𝜃 = 6 − 𝜋 𝑜𝑟 𝜃 = − 6 , that is,
5𝜋 𝜋
𝜃=− 𝑜𝑟 𝜃 = −
6 6

You can easily verify on your calculator that taking the sine of both angles gives -0.5 as
required.

In this example, we have restricted our range of solutions to the range −𝜋 ≤ 𝜃 ≤ 𝜋. If we


remove this restriction, our solution set for the above problem will now be represented by the
following graph:

55
You can clearly see our 2 solutions in the range [−𝜋; 𝜋] where the 2 graphs intersect.
However, the two graphs intersect each other several times, in fact, an infinite number of
times the only limitation being the amount of space of your page. This means there are an
infinite number of solutions for the function sin 𝑥 = −0.5. However, there is a simple way of
generalising these solutions simply by realising that the sine graph repeats a solution after
every full cycle of 2𝜋. These solutions range from negative to positive infinity therefore all
solutions are integer multiples of 2𝜋.
Therefore, in light of this discussion, it means that the general solution of
sin 𝑥 = −0.5 is given by

5𝜋 𝜋
𝜃=− + 2𝜋𝑘 or θ=− + 2𝜋𝑘; 𝑘∈𝑍
6 6

Remember integers are positive and negative whole numbers and zero so it is sufficient to
write +2𝜋𝑘 in the general solutions and not ±2𝜋𝑘. The procedure of obtaining the result for
the cosine function is the same.

Let’s together go through an example to cement our understanding of the general


solution.

Question
Find the general solution of the following equation:
𝜋
cos �2𝜋𝑥 − � = −1
3

Solution
Cosine is negative in the 2nd and 3rd quadrants, therefore
𝛼 = cos −1 1 = 0

56
𝜋 𝜋
→ 2𝜋𝑥 −
= 𝜋 − 0 + 2𝜋𝑘 or 2𝜋𝑥 − = 0 − 𝜋 + 2𝜋𝑘
3 3
𝜋 𝜋
→ 2𝜋𝑥 − = 𝜋 + 2𝜋𝑘 or 2𝜋𝑥 − = −𝜋 + 2𝜋𝑘
3 3
𝜋 𝜋
→ 2𝜋𝑥 = + 𝜋 + 2𝜋𝑘 or 2𝜋𝑥 = − 𝜋 + 2𝜋𝑘
3 3
4𝜋 2𝜋
→ 2𝜋𝑥 = + 2𝜋𝑘 or 2𝜋𝑥 = − + 2𝜋𝑘
3 3
2 1
∴ 𝑥 = + 𝑘 or 𝑥 = − + 𝑘; 𝑘 ∈ 𝑍
3 3

Turn to page 25 of the study guide and attempt the whole of Question 4.

REFLECTION
In this section, we have seen how we can solve trigonometric equations using graphs. When
solving, remember you want the value of x, which you should read off from the x-axis. If you
use a calculator when you have been asked to use a graph, you gain no credit, always follow
the given instruction. The solution using graphical means should be indicated on the graph.
We also covered the general solution of a trig function. Since a trig function is periodic, there
is an infinite number of solutions and this leads to the general solution of a trig function.

8.3 Test 1
At this point, you have now covered enough content to sit for Mathematics 1 test 1,
congratulations. It is that time where you get to go over all content covered and get yourself
ready for the test. Pay particular attention to the common mistakes addressed so far so that
you don’t become a statistic as well. Iron out any questions with your tutor or you can come
into my office anytime for consultation.

Good luck and let’s meet in the next content of Differential Calculus where we will explore
differentiation and integration.

Please consult with your lecturers on the scope for test 1 as it sometimes changes due to
circumstances beyond our control like student unrest etc.

57
CONTENT AREA 3: DIFFERENTIAL CALCULUS

DIFFERENTIAL CALCULUS

Standard Derivatives

Product Rule

Quotient Rule

Chain Rule

Implicit Differentiation

Logarithmic Differentiation

Higher Order Derivatives

Applications: Maxima and Minima; equations of tangents


and normals; motion

WARNING: THIS IS BY FAR THE LONGEST CONTENT AREA AND YOU ARE
EXPECTED TO DEVOTE CONSIDERABLE RESOURCES AND EFFORT ON THIS
AREA. THE WORK YOU DO NOW WILL LAY A FOUNDATION FOR VIRTUALLY
THE ENTIRE MATHEMATICS 2 COURSE AS 60% WILL BE USED DIRECTLY
AND INDIRECTLY. MANY STUDENTS FIND THIS AREA CHALLENGING AND
PERFORMING POORLY, ESPECIALLY THOSE WHO WOULD HAVE ATTAINED
HIGH MARKS IN TEST 1. THE MAIN REASON IS INDISCIPLINE AS STUDENTS
MISS SEVERAL LECTURES AND TUTORIALS AFTER A GOOD
PERFORMANCE IN TEST 1 AND SOME WILL BE SACRIFICING MATHS 1 DUE
TO PRESSURES FROM OTHER MODULES. BOTH STRATEGIES WILL SET YOU
FOR CERTAIN DISAPPOINTMENT. PASSING A MATHEMATICS COURSE
REQUIRES ‘A CONSISTENT AND SUSTAINED EFFORT’ AS PROF M
RAMOGALE WOULD PUT IT NICELY. DON’T BE CAUGHT UP IN THIS FALSE
SENSE OF COMPLACENCY.

REMEMBER YOU HAVE BEEN WARNED!

58
9 Differentiation – Basic
Learning Outcomes
After completing this section, you should be able to
• Find the derivatives of algebraic, transcendental, and trigonometric functions using
standard derivatives

9.1 Introduction to differentiation


The process of finding the general expression for the gradient of a curve is known as
differentiation. The general gradient expression for a curve 𝑦 = 𝑓(𝑥) is itself a function so it
is called the gradient function. For the curve 𝑦 = 𝑥 2 for example, the gradient function is 2𝑥.
Because the gradient function is derived from the given function, it is more often called the
derivative or more commonly, the derivative.
Differentiation Notation
Differentiating 𝑥 2 gives 2𝑥. One way to write this fact, based on the equation of the curve, is
𝑑𝑦
for the curve 𝑦 = 𝑥 2 , = 2𝑥 (we say d𝑦 by d𝑥)
𝑑𝑥

Note carefully that d has no independent meaning and must never be regarded as a factor.

𝑑
The complete symbol 𝑑𝑥 means ‘the derivative of ⋯ with respect to 𝑥’
𝑑𝑦
Therefore, 𝑑𝑥 means ‘the derivative of y with respect to x’ and
𝑑
(𝑥 2 − 𝑥) means ‘the derivative of (𝑥 2 − 𝑥) with respect to 𝑥.
𝑑𝑥
An alternative notation concentrates on the function of x rather than the equation of the curve.
An example is
for 𝑓(𝑥) = 𝑥 2 , 𝑓 ′ (𝑥) = 2𝑥
In this form, 𝑓 ′ means ‘the gradient function’ or ‘the derived function’.
Either of these notations can be used for variables other than 𝑥, e.g.
if 𝑦 = 𝑧 3 then we can differentiate 𝑦 with respect to 𝑧
𝑑𝑦
and write = 3𝑧 2
𝑑𝑧
Similarly, if 𝑠 = 𝑡 2 − 𝑡, we differentiate 𝑠 with respect to 𝑡
𝑑𝑠
and write = 2𝑡 − 1
𝑑𝑡
(Because the phrase ‘with respect to’ is used very frequently, it is often abbreviated to w.r.t.)

Mistake 13
𝑑𝑦
Taking 𝑑𝑥 to mean the derivative in every function. Read the last paragraph above, use the
appropriate variables when writing the derivative notation. You may be penalised if you use
𝒅𝒚 𝒅𝒔
say 𝒅𝒙 when you were supposed to use the notation say 𝒅𝒕. The same applies if you always
refer to the derivative as 𝑓 ′ (𝑥) when say you were supposed to use 𝑠 ′ (𝑡).

59
9.2 Differentiating 𝒙𝒏 with respect to 𝒙
If 𝑦 = 𝑥 𝑛 , then
𝑑𝑦
= 𝑛𝑥 𝑛−1
𝑑𝑥

Example
Differentiate w.r.t. x the following
a) 𝑦 = 1⁄𝑥
1
b) 𝑦 = 𝑥 −2
c) 𝑦 = √𝑥 7
d) 𝑦 = √(𝑥 2 )3

Solution
a)
𝑦 = 1�𝑥 = 𝑥 −1
𝑑𝑦 1
→ = (−1)𝑥 −2 = − 2
𝑑𝑥 𝑥
b)
1
𝑦 = 𝑥 −2
𝑑𝑦 1 3 1
→ = �− � 𝑥 −2 = − 2
𝑑𝑥 2 2 √𝑥 3
c)
7
𝑦 = �𝑥 7 = 𝑥 2
𝑑𝑦 7 5 7√𝑥 5
→ = � � 𝑥2 =
𝑑𝑥 2 2
d)
𝑦 = �(𝑥 2 )3 = �𝑥 6 = 𝑥 3
𝑑𝑦
→ = 3𝑥 2
𝑑𝑥

NB: The section on differential calculus assumes a mastery of the laws of indices and
logarithms. If you feel rusty on those laws, my suggestion is that you go back and revise
them immediately as they will be applied from now on without further reference.

Differentiate the following w.r.t. x.


a) 𝑦 = 𝑥5
b) 𝑦 = 1⁄𝑥 4
c) 𝑦 = √𝑥 3
d) 𝑦 = ∛𝑥 2
e)

60
9.3 Differentiating a constant
The equation of a straight line 𝑦 = 𝑐 is a horizontal straight line parallel to the 𝑥 − axes. Its
𝑑𝑦
gradient is therefore zero, that is, 𝑑𝑥 = 0.
𝑑𝑦
if 𝑦 = 𝑐 then =0
𝑑𝑥

Example
Students sometimes miss out on the following constants whose derivatives are zero
𝑦 = 𝜋2
𝑑𝑦
→ =0
𝑑𝑥
𝑦 = 𝑒3
𝑑𝑦
=0
𝑑𝑥
𝑒 and π are constants and their derivatives and those of their powers, are always zero.

9.4 Differentiating a linear function of x


If 𝑎 is a constant,
𝑑
𝑎𝑥 𝑛 = 𝑎𝑛𝑥 𝑛−1
𝑑𝑥

Example
𝑦 = 3𝑥 5
𝑑𝑦
→ = 3 × 5𝑥 4 = 15𝑥 4
𝑑𝑥
4
𝑦 = 2 = 4𝑥 −2
𝑥
𝑑𝑦 8
→ = 4 × (−2)𝑥 −3 = −8𝑥 −3 = − 3
𝑑𝑥 𝑥

Another very useful property is


a function of 𝑥 which contains a number of different terms can be differentiated term by term,
applying the basic rule to each term in turn.

Example
Differentiate the following functions w.r.t. x
a) 𝑦 = 𝑥 3 − 𝑥 2 + 5𝑥 − 6
1 1
b) 𝑦 = 𝑥 2 − 𝑥 3
c) 𝑦 = 3√𝑥 − 3𝑥
d) 𝑦 = 𝑥 − 2𝑥 −1 − 3𝑥 −3

Solution
a)
𝑦 = 𝑥 3 − 𝑥 2 + 5𝑥 − 6
𝑑𝑦
→ = 3𝑥 2 − 2𝑥 + 5
𝑑𝑥

61
b)
1 1
𝑦= − = 𝑥 −2 − 𝑥 −3
𝑥2 𝑥3
𝑑𝑦 2 3
→ = (−2)𝑥 −3 − (−3)𝑥 −4 = − 3 + 4
𝑑𝑥 𝑥 𝑥
c)
1
𝑦 = 3√𝑥 − 3𝑥 = 3𝑥 2 − 3𝑥
𝑑𝑦 1 1 3
→ = 3 × � � 𝑥 −2 − 3 = −3
𝑑𝑥 2 2 √𝑥
d)
𝑦 = 𝑥 − 2𝑥 −1 − 3𝑥 −3
𝑑𝑦 2 9
→ = 1 + 2𝑥 −2 + 9𝑥 −4 = 1 + 2 + 4
𝑑𝑥 𝑥 𝑥

9.5 Other common derivatives


We now briefly outline other important derivatives that you will utilise in this module.
Derivative of 𝒚 = 𝐬𝐢𝐧 𝒙
𝑑𝑦
If 𝑦 = sin 𝑥, = cos 𝑥
𝑑𝑥

Derivative of 𝒚 = 𝐜𝐨𝐬 𝒙
𝑑𝑦
If 𝑦 = cos 𝑥, = − sin 𝑥
𝑑𝑥

Derivative of 𝒚 = 𝐭𝐚𝐧 𝒙
𝑑𝑦
If 𝑦 = tan 𝑥 , = sec 2 𝑥
𝑑𝑥

Derivative of 𝒚 = 𝒆𝒙
𝑑𝑦
If 𝑦 = 𝑒 𝑥 , = 𝑒𝑥
𝑑𝑥

Derivative of 𝒚 = 𝐥𝐧 𝒙
𝑑𝑦 1
If 𝑦 = ln 𝑥 , =
𝑑𝑥 𝑥

The list goes on and on. You are not expected to memorise these derivatives but you are
expected to know when to apply them especially in light of the following rules that I will
look into next. We will encounter more standard derivatives as we go.

REFLECTION
In this section, we covered the introduction to differentiation. The derivative is a function for
the slope of a curve and the derivative for a function y which is a function of x, we denote the
62
dy
derivative by . We went through several standard derivatives that you will meet in this
dx
course. The derivative of x n is nx n −1. and the derivative of a polynomial is the derivative sum
of the derivative of the individual terms. There is no need to memorise the derivatives as the
standard derivatives are supplied in a test and/or exam.

63
10 Differentiation of Compound Functions
Learning Outcomes
After completing this section, you should be able to
• Find derivatives using the product, quotient and chain rules

10.1 Differentiation of a product of functions


A product of functions results when two functions are multiplying each other. Examples of
products of functions are

𝑦 = 𝑥 3 sin 𝑥 , 𝑦 = 𝑥 5 𝑒 𝑥 , 𝑦 = 𝑥 ln 𝑥, 𝑦 = (𝑥 2 + 4𝑥)(𝑥 3 + 7𝑥 2 + 5𝑥 + 1), 𝑦 = 𝑥 tan 𝑥

To differentiate a product of two functions, 𝑦 = 𝑢𝑣 where both u and v are functions of x,


we use the product rule which is given as follows:
(𝑢𝑣)′ = 𝑢′ 𝑣 + 𝑢𝑣 ′

Example
𝑑𝑦
The product rule can be illustrated by taking two examples from the above. Find 𝑑𝑥 of the
following functions
a) 𝑦 = 𝑥 5 𝑒 𝑥
b) 𝑦 = 𝑥 ln 𝑥

Solution
a)
𝑦 = 𝑥5𝑒 𝑥
𝑑𝑦
→ = 5𝑥 4 𝑒 𝑥 + 𝑥 5 𝑒 𝑥
𝑑𝑥

b)
𝑦 = 𝑥 ln 𝑥
𝑑𝑦 1
→ = ln 𝑥 + 𝑥 ∙ = ln 𝑥 + 1
𝑑𝑥 𝑥

Mistake 14
Inappropriate use of the product rule to differentiate a ‘product’ of a function and a constant.

Example
𝑑𝑦
Find 𝑑𝑥 of the following functions
a) 𝑦 = 3 ln 𝑥
b) 𝑦 = 𝜋 3 𝑥
c) 𝑦 = 𝑒 2 𝑥 5

Typical student solutions


a)
𝑦 = 3 ln 𝑥
𝑑𝑦 1 3
→ = 0 ∙ ln 𝑥 + 3 ∙ =
𝑑𝑥 𝑥 𝑥

64
Comment: This solution may be correct but the conceptualisation is definitely wrong. 3 ln 𝑥
is not a product of two functions of x, rather, since 3 is a constant, it is only a coefficient of ln
x and so to differentiate the function, you simply need to differentiate it as a linear function,
that is,
𝑦 = 3ln 𝑥
𝑑𝑦 1 3
→ = 3∙ =
𝑑𝑥 𝑥 𝑥

b)
𝑦 = 𝜋3𝑥
𝑑𝑦
→ = 3𝜋 2 𝑥 + 𝜋 3 ∙ 1 = 3𝜋 2 𝑥 + 𝜋 3
𝑑𝑥

Comment: The solution is horribly wrong here and it emanates from the thinking of treating
𝜋 (and all its powers) as a function of x instead of a constant. Again, the differentiation
approach is similar to the one in (a) where you simply differentiate it as a linear function to
give

𝑦 = 𝜋3𝑥
𝑑𝑦
→ = 𝜋3
𝑑𝑥

Period! Always leave the answer in terms of 𝝅, you will be penalised if you use a
calculator to evaluate the value of 𝝅𝟑 .

c)
𝑦 = 𝑒2𝑥5
𝑑𝑦
→ = 2𝑒𝑥 5 + 5𝑒 2 𝑥 4
𝑑𝑥

Comment: The mistake here is almost the same as the one in (b), e is being treated like x.
For the record, e (and all its powers) is a constant. 𝑒 2 is just a coefficient of 𝑥 5 and so the
product rule should not be applied. Instead, the correct solution should be as follows:

𝑦 = 𝑒2𝑥5
𝑑𝑦
→ = 𝑒 2 ∙ 5𝑥 4 = 5𝑒 2 𝑥 4
𝑑𝑥

𝑑𝑦
Find 𝑑𝑥 of the following functions using the product rule:
a)
𝑦 = 𝑥 3 sin 𝑥
b)
𝑦 = 𝑥 tan 𝑥
c)
𝑦 = (2𝑥 2 + 1)(2𝑥 − 5)

65
10.2 Differentiation a quotient
𝑢
Consider a function 𝑦 = 𝑣 , where both u and v are functions of x. To differentiate this
function, we use the quotient rule which is stated as follows:

𝑢 ′ 𝑣𝑢′ − 𝑢𝑣 ′
� � =
𝑣 𝑣2

Example
𝑑𝑦
Find 𝑑𝑥 for the following functions using the quotient rule.
a)
sin 𝑥
𝑦= 2
𝑥
b)
ln 𝑥 3
𝑦=
𝑥
Solution
a)
sin 𝑥
𝑦= 2
𝑥
By the Quotient Rule,
𝑑𝑦 𝑥 2 ∙ cos 𝑥 − sin 𝑥 ∙ 2𝑥 𝑥 2 cos 𝑥 − 2𝑥 sin 𝑥 𝑥 cos 𝑥 − 2 sin 𝑥
= = =
𝑑𝑥 (𝑥 2 )2 𝑥4 𝑥3

Mistake 15
Giving an answer which is not in its lowest terms. Look at (a) above, in the final solution, x is
cancelled out from the denominator and the numerator because it is a common factor. Always
look for common factors so that you cross them out, this extends to both numbers and
algebraic terms.

You will be penalised if you don’t cancel out common factors to give a solution in lowest
terms.

b)
ln 𝑥 3 3 ln 𝑥
𝑦= =
𝑥 𝑥
1
𝑑𝑦 𝑥 ∙ − ln 𝑥 ∙ 1 1 − ln 𝑥 3(1 − ln 𝑥)
→ = 3� 𝑥 2 � = 3� � =
𝑑𝑥 𝑥 𝑥2 𝑥2

10.3 Differentiating a function of a function


Suppose we want to differentiate 𝑦 = (𝑥 2 + 2)3. We could expand the bracket (with lots of
hard labour) and differentiate term by term, but this is tedious and for powers higher than
three, very long and not easy. We obviously need a more direct method for differentiating an
expression of this kind. To do this, we take (𝑥 2 + 2)3 as a composite or combined function.

66
Now (𝑥 2 + 2)3 is a cubic function of the quadratic function (𝑥 2 + 2), that is, it is a function
of a function.

A function of this type is of the form 𝑓𝑔(𝑥), that is 𝑓�𝑔(𝑥)� where 𝑓(𝑥) = 𝑥 3 and 𝑔(𝑥) =
𝑥 2 + 2.

Consider any function of the form 𝑦 = 𝑓𝑔(𝑥)


If we make the substitution 𝑢 = 𝑔(𝑥), then 𝑦 = 𝑓𝑔(𝑥) can be expressed in two simple parts,
that is
𝑢 = 𝑔(𝑥) and 𝑦 = 𝑓(𝑢)
To evaluate the derivative of a function of a function 𝑦 = 𝑓𝑔(𝑥) = 𝑓(𝑢), we use the
following formula

𝑑𝑦 𝑑𝑦 𝑑𝑢
= ×
𝑑𝑥 𝑑𝑢 𝑑𝑥

This is known as the chain rule.

NB: du cannot be cancelled on the right hand side as these are not fractions but derivatives.
However, the notation suggests the result and is easy to remember.

Example
𝑑𝑦
Find 𝑑𝑥 given that 𝑦 = (𝑥 2 + 2)3

Solution
Take 𝑢 = 𝑥 2 + 2 as the core. Then 𝑦 = 𝑢3 .
Therefore
𝑑𝑢
= 2𝑥
𝑑𝑥
and
𝑑𝑦
= 3𝑢2
𝑑𝑢
and so
𝑑𝑦 𝑑𝑦 𝑑𝑢
= × = 3𝑢2 × (2𝑥) = 3(𝑥 2 + 2)2 × (2𝑥) = 6𝑥(𝑥 2 + 2)2
𝑑𝑥 𝑑𝑢 𝑑𝑥

NB: With practice, this step will be performed mentally with no loss of generality.

One thing you should learn to appreciate about differentiation is that all of the rules
we have looked at can work together in the same question! Never take a ‘one-dimensional’
approach to solving differentiation questions; you need to bring all your skills together
especially those you learnt from laws of indices and logarithms. As you will probably
imagine, some of the substitutions which you have to do may have to be relegated to rough
work. I will gradually drop out explicitly stating some substitutions but in way that you are
able to follow. The same thinking should follow you when you answer tests/exams. You can
skip some introductory steps but rest assured, the marker will be able to follow, they are
mathematicians after all.

67
Let’s go through the following examples. I will leave out some of the ‘flesh’ and you should
fill in the gaps if you don’t understand.

Questions
Differentiate the following
a)
𝑦 = 2𝑥 cos 𝑥 − 2 sin 𝑥

b)
𝑥
𝑦= 3
√𝑥 2 + 4

c)
𝑦 = 𝑥 ln sin 𝑥

d)
𝑦 = cos 2 (2𝑥 + 1)

Solution
a) = y 2 x cos x − 2sin x
dy
⇒ =
2 cos x − 2 x sin x − 2 cos x =
−2 x sin x
dx

NB: You should be able to notice that I have applied the product rule.
a)
𝑥 𝑥
𝑦=3 = 1
√𝑥 2 + 4 (𝑥 2 + 4)3
2
1 1 2 −2
𝑑𝑦 (𝑥 + 4)3 ∙ 1 − 𝑥 ∙ 3 (𝑥 + 4) 3 ∙ 2𝑥
→ = 2
𝑑𝑥 (𝑥 2 + 4)3

NB: I have to pause and make a comment here. Many students get to this point without
difficulty but are at loss as to where to go from here. I will start by rewriting the above
derivative in terms of positive powers on and multiply throughout to clear the fractional part
from the numerator. This means
1 2𝑥 2
�(𝑥 2 + 4)3 − 2�
𝑑𝑦 3(𝑥 2 + 4)3
= 2
𝑑𝑥 (𝑥 2 + 4)3

The square brackets in the numerator are there for clarity


1 2𝑥 2 2
�(𝑥 2 + 4)3 − 2
2 � × 3(𝑥 + 4)3
𝑑𝑦 3(𝑥 2 + 4)3
→ = 2 2
𝑑𝑥 (𝑥 2 + 4)3 × 3(𝑥 2 + 4)3
𝑑𝑦 3(𝑥 2 + 4) − 2𝑥 2
→ = 4
𝑑𝑥 (𝑥 2 + 4)3
𝑑𝑦 3𝑥 2 + 12 − 2𝑥 2
→ = 4
𝑑𝑥 (𝑥 2 + 4)3

68
𝑑𝑦 𝑥 2 + 12
→ =
𝑑𝑥 3�(𝑥 2 + 4)4

NB: Your mastery of the laws of indices as well as fractions is expected in this question.

b)
𝑦 = 𝑥 ln sin 𝑥
Approach: Product rule and chain rule
𝑑𝑦 1
→ = ln sin 𝑥 + 𝑥 � ∙ cos 𝑥�
𝑑𝑥 sin 𝑥
𝑑𝑦 cos 𝑥
→ = ln sin 𝑥 + 𝑥
𝑑𝑥 sin 𝑥
𝑑𝑦
→ = ln sin 𝑥 + 𝑥 cot 𝑥
𝑑𝑥

NB: Knowledge and application of basic trig identities is expected here

c)
𝑦 = cos 2 (2𝑥 + 1)
𝑑𝑦
→ = 2 cos(2𝑥 + 1) ∙ (− sin(2𝑥 + 1)) ∙ 2𝑥
𝑑𝑥
𝑑𝑦
→ = −4𝑥 cos(2𝑥 + 1) sin(2𝑥 + 1)
𝑑𝑥
d)
𝑦 = 𝑒 sin 𝑥 + 𝑥 2𝑒 + sin 𝑒 + tan 𝑒𝑥
𝑑𝑦
→ = cos 𝑥𝑒 sin 𝑥 + 2𝑒𝑥 2𝑒−1 + 0 + sec 2 𝑒𝑥 ∙ 𝑒
𝑑𝑥
𝑑𝑦
→ = cos 𝑥𝑒 sin 𝑥 + 2𝑒𝑥 2𝑒−1 + 𝑒sec 2 𝑒𝑥
𝑑𝑥
NB: e is a constant and so is 2e and sin e.

10.4 Use of standard derivatives formulae


To this end, I haven’t raised any discussion on the use of the standard formula. The reason for
that is the fact that you needed to understand the chain rule first before knowing how to use
the formulae properly. The derivatives are given ‘in chain rule format’ but this should not
confuse you. In fact, it serves as an added advantage to the fact that the required substitutions
have already been done for you, all you need to do is identify the variables. I will begin by
tackling the issue of how to tackle the standard derivatives of functions like sin x, cos x, tan
x, ln x, ax, to name but a few.

Turn to the study guide pages 39-40. On these pages, you will see all the standard derivatives
that you are supposed to use in this course. You are not expected to memorise any of the
derivatives as the formulae will be supplied to you in a test or examination. But practice
helps you extract the required derivatives quickly and accurately.

69
Suppose you want to look up the derivative of sin x from the list of formula. The 6th
derivative formula is written as

𝑑
[sin 𝑓(𝑥)] = 𝑓 ′ (𝑥) cos 𝑓(𝑥)
𝑑𝑥

To get the derivative of sin x, simply set 𝑓(𝑥) = 𝑥 which implies 𝑓 ′ (𝑥) = 1, reducing the
above formula to
𝑑
[sin 𝑥] = 1 ∙ cos 𝑥 = cos 𝑥
𝑑𝑥

And there you have the required derivative. Likewise, if you want to get the derivative of tan
x, you say from formula 8:

𝑓(𝑥) = 𝑥
→ 𝑓 ′ (𝑥) = 1
𝑑
[tan 𝑥] = sec 2 𝑥
𝑑𝑥

Another example is the derivative of 𝑎 𝑥 . From formula 3, we have

𝑓(𝑥) = 𝑥
→ 𝑓(𝑥) = 1
𝑑 𝑥
∴ [𝑎 ] = 𝑎 𝑥 ln 𝑎
𝑑𝑥

I hope by now you get the picture that is emerging. Now, let’s go back to the chain rule itself
and work out a few functions by using the standard derivatives.

Example
Find the derivatives of the following functions
a) 𝑦 = ln sin 𝑥
b) 𝑦 = 𝑒 sin 𝑥
c) 𝑦 = tan 𝑒𝑥
d) 𝑦 = ln(𝑥 2 + 1)
e) 𝑦 = log 7 3𝑥 2
f) 𝑦 = cosec 𝑒𝑥
g) 𝑦 = sec 3 𝑥
h) 𝑦 = (3𝑥 + 7)2

Solution
For each of the solutions, I will be referring you to the formula to which I have quoted the
derivative from so that you can also practice the ‘art’ of looking up the derivatives yourself.

a) Formula no. 4
𝑓(𝑥) = sin 𝑥 → 𝑓 ′ (𝑥) = cos 𝑥
𝑑𝑦 𝑓 ′ (𝑥) cos 𝑥
∴ = = = cot 𝑥
𝑑𝑥 𝑓(𝑥) sin 𝑥

70
b) Formula no. 2
𝑓(𝑥) = sin 𝑥 → 𝑓 ′ (𝑥) = cos 𝑥
𝑑𝑦
∴ = 𝑓 ′ (𝑥)𝑒 𝑓(𝑥) = cos 𝑥𝑒 sin 𝑥
𝑑𝑥

c) Formula no. 8
𝑓(𝑥) = 𝑒𝑥 → 𝑓 ′ (𝑥) = 𝑒
𝑑𝑦
∴ = 𝑓 ′ (𝑥)sec 2 𝑓(𝑥) = 𝑒sec 2 𝑒𝑥
𝑑𝑥

d) Formula no. 4
𝑓(𝑥) = 𝑥 2 + 1 → 𝑓 ′ (𝑥) = 2𝑥
𝑑𝑦 𝑓 ′ (𝑥) 2𝑥
∴ = = 2
𝑑𝑥 𝑓(𝑥) 𝑥 + 1

e) Formula no. 5
𝑓(𝑥) = 3𝑥 2 → 𝑓 ′ (𝑥) = 6𝑥
𝑑𝑦 𝑓 ′ (𝑥) 6𝑥 2
∴ = = 2 =
𝑑𝑥 𝑓(𝑥) ln 𝑎 3𝑥 ln 7 𝑥 ln 7

f) Formula no. 11
𝑓(𝑥) = 𝑒𝑥 → 𝑓 ′ (𝑥) = 𝑒
𝑑𝑦
∴ = −𝑓 ′ (𝑥)cosec𝑓(𝑥) cot 𝑓(𝑥) = −𝑒cosec 𝑒𝑥 cot 𝑒𝑥
𝑑𝑥

g) Formula no 1
𝑓(𝑥) = sec 𝑥 → 𝑓 ′ (𝑥) = sec 𝑥 tan 𝑥
𝑑𝑦
∴ = 𝑛[𝑓(𝑥)]𝑛−1 𝑓 ′ (𝑥) = 3sec 2 𝑥 ∙ sec 𝑥 tan 𝑥 = 3sec 3 𝑥 tan 𝑥
𝑑𝑥

h) Formula no. 1
𝑓(𝑥) = 3𝑥 + 7 → 𝑓 ′ (𝑥) = 3
𝑑𝑦
∴ = 𝑛[𝑓(𝑥)]𝑛−1 𝑓 ′ (𝑥) = 2(3𝑥 + 7) ∙ 3 = 6(3𝑥 + 7)
𝑑𝑥

So far so good hey? I am sure you are also noticing that some of the substitutions can be done
mentally and that is perfectly permissible, in fact, I encourage you reduce the steps with time.
You rarely get a simple function like the ones above, I only wanted to get you into the correct
tune for using the given standard derivatives.

Derivative of a function of a function of a function of a function….

Example
Consider a function like
2 +5
𝑦 = sin2 �𝑒 √𝑥 �

Using the technique in the previous example, we have

71
2 +5
2
𝑦 = �sin �𝑒 √𝑥 ��

So by Formula no. 1, we have


𝑑𝑦 2 2

= 𝑛[𝑓(𝑥)]𝑛−1 𝑓 ′ (𝑥) = 2 sin �𝑒 √𝑥 +5 � ∙ �sin �𝑒 √𝑥 +5 ��
𝑑𝑥

By Formula no. 6, we have


𝑑𝑦 2 2 2

= 2 sin �𝑒 √𝑥 +5 � ∙ cos �𝑒 √𝑥 +5 � ∙ �𝑒 √𝑥 +5 �
𝑑𝑥

By Formula 2, we have
𝑑𝑦 2 2 2

= 2 sin �𝑒 √𝑥 +5 � ∙ cos �𝑒 √𝑥 +5 � ∙ 𝑒 √𝑥 +5 ∙ ��𝑥 2 + 5�
𝑑𝑥

By Formula no. 1, we have


𝑑𝑦 2 2 2 1 1
= 2 sin �𝑒 √𝑥 +5 � ∙ cos �𝑒 √𝑥 +5 � ∙ 𝑒 √𝑥 +5 ∙ � (𝑥 2 + 5)−2 ∙ 2𝑥�
𝑑𝑥 2

Normally, it is not necessary to simplify so at this stage we can say we have achieved our
objective of differentiating. What has been going on is repeated chain rule application until
we finally reached the simply function right at the core. At each stage, you should look up the
relevant derivative and when you do, see if it needs to be repeated again going on until you
arrived at the answer. Let’s look at another example so that your questions are quickly
cleared

Example
Find the derivative of the following functions
a)
𝑦 = 𝑒 cos(2𝑥+1)
b)
𝑦 = �tan 𝑥 2
c)
𝑦 = tan3 𝑥 5

Solution
Like the previous example, I will work out the question in stages but this time I will leave
you to figure out the formula numbers as I expect that you should be familiar with them by
now from your lectures.

a)
𝑦 = 𝑒 cos(2𝑥+1)
𝑑𝑦
→ = 𝑒 cos(2𝑥+1) ∙ (cos(2𝑥 + 1))′
𝑑𝑥
𝑑𝑦
→ = 𝑒 cos(2𝑥+1) ∙ (− sin(2𝑥 + 1)) ∙ (2𝑥 + 1)′
𝑑𝑥
𝑑𝑦
→ = 𝑒 cos(2𝑥+1) ∙ (− sin(2𝑥 + 1)) ∙ 2
𝑑𝑥
𝑑𝑦
∴ = −2 sin(2𝑥 + 1)𝑒 cos(2𝑥+1)
𝑑𝑥

72
Nothing to explain here, let’s move to the next problem.

b)
1
𝑦 = �tan 𝑥 2 = (tan 𝑥 2 )2
𝑑𝑦 1 1
= (tan 𝑥 2 )−2 ∙ (tan 𝑥 2 )′
𝑑𝑥 2
𝑑𝑦 1 1
→ = (tan 𝑥 2 )−2 ∙ sec 2 𝑥 2 ∙ (𝑥 2 )′
𝑑𝑥 2
𝑑𝑦 1 1
→ = (tan 𝑥 2 )−2 ∙ sec 2 𝑥 2 ∙ 2𝑥
𝑑𝑥 2
𝑑𝑦 𝑥sec 2 𝑥 2
∴ =
𝑑𝑥 √tan 𝑥 2

I hope you are getting used by now. Let’s finish off the last one.

c)
𝑦 = tan3 𝑥 5 = [tan 𝑥 5 ]3
𝑑𝑦
= 3[tan 𝑥 5 ]2 ∙ (tan 𝑥 5 )′
𝑑𝑥
𝑑𝑦
→ = 3[tan 𝑥 5 ]2 ∙ sec 2 𝑥 5 ∙ (𝑥 5 )′
𝑑𝑥
𝑑𝑦
→ = 3[tan 𝑥 5 ]2 ∙ sec 2 𝑥 5 ∙ 5𝑥 4
𝑑𝑥
𝑑𝑦
∴ = 15𝑥 4 sec 2 𝑥 5 [tan 𝑥 5 ]2
𝑑𝑥

I am sure you are getting tempted into solving evaluating these derivatives in 2 lines. If you
can, it’s alright but do not be pressured into doing so. Following this kind of a build-up
allows you to minimise the risk of unnecessary errors during calculation.

Mistake 16
Find the derivative of the following question
𝑦 = ln(𝑥 2 + 1)3

Typical student answer


𝑑𝑦 ((𝑥 2 + 1)3 )′ 3(𝑥 2 + 1)2 ∙ 2𝑥 6𝑥
= = =
𝑑𝑥 (𝑥 2 + 1)3 (𝑥 2 + 1)3 𝑥2 + 1

What’s wrong here? Be the judge, from the alternative solution below

𝑦 = ln(𝑥 2 + 1)3 = 3 ln(𝑥 2 + 1)


𝑑𝑦 2𝑥 6𝑥
= 3∙ 2 = 2
𝑑𝑥 𝑥 +1 𝑥 +1

COMMENT: You gain no honour by engaging an army to kill a mouse.

73
Mistake 17
Find the derivative of the following function
𝑦 = 𝑒 2 ln 𝑥

Typical student solution


𝑑𝑦 2
= 𝑒 2 ln 𝑥 ∙ (2 ln 𝑥)′ = 𝑒 2 ln 𝑥
𝑑𝑥 𝑥

COMMENT: You won’t get full marks.

Alternative solution
2
𝑦 = 𝑒 2 ln 𝑥 = 𝑒 ln 𝑥 = 𝑥 2
𝑑𝑦
∴ = 2𝑥
𝑑𝑥

COMMENT: Always be alert and never think in one dimension. You are allowed and
encouraged to simplify an expression where possible before differentiating.

It has been a lot of work in this section and it’s time to work out a few problems to
see how much you are gaining on this material. But it surely is an interesting topic.

Turn to page 26 of the study guide and do Question 1 and Question 2 (page 27) of Tutorial 3.
We now move on to implicit and logarithmic differentiation in the next section.

REFLECTION
In this section, we went further to different compound functions.
The product rule is used to find the derivative of a product of two functions of x, eg, if
y ( x ) = f ( x ) g ( x ) , it implies that
=y ' f ' g + fg '
This implies that if y = x sin x, 2

d ( x 2 sin x )
= 2 x sin x + x 2 cos x
dx
f
The quotient rule is used to find the derivative of a quotient of two functions of x. If y = ,
g
it implies that
gf '− fg '
y '( x) =
g2
sin x
by the quotient rule. For example, if y =
, it implies that
x2
x 2 cos x − sin x ⋅ ( 2 x ) x 2 cos x − 2 x sin x
=y' =
( x2 )
2
x4
The quotient rule is used to evaluate the derivative of a compound function, or a function of a
=
function. If y f= ( g ( x ) ) f ( g ) , it implies that

74
dy df dg
= × .
dx dg dx
y sin ( x 2 + 3 x + 2 ) , g ( x ) = x 2 + 3 x + 2 and f ( x ) = sin x. Since y = f ( g ) ,
For example, if =
it implies that
= cos ( x 2 + 3 x + 2 ) ⋅ ( 2 x + 3) = ( 2 x + 3) cos ( x 2 + 3 x + 2 )
dy
dx
The chain rule can also be extended to a function of a function of a function. For example, if
=y ln sin ( x 2 + 3 x + 2 ) , it implies that

⋅ cos ( x 2 + 3 x + 2 ) ⋅ ( 2 x + 3) = ( 2 x + 3) cot ( x 2 + 3x + 2 )
dy 1
=
dx sin ( x + 3 x + 2 )
2

This chain rule can even be extended further and is only limited by one’s imagination.

75
11 Differentiating Implicit Functions
Learning Outcomes
After completing this section, you should be able to
• Find derivatives of implicit functions
• Find derivatives using logarithmic differentiation

11.1 Implicit Functions


All the differentiation carried out so far has involved equations that could be expressed in the
form 𝑦 = 𝑓(𝑥), that is, 𝑦 is expressed explicitly.

However, the equation of some curves, for example 𝑥 2 − 𝑦 2 + 𝑦 = 1, cannot be easily


written in this way, as it is too difficult to isolate y.

A relationship of this type, where y is not given explicitly as a function of x, is called an


implicit function, that is, it is implied in the equation that 𝑦 = 𝑓(𝑥).

11.2 Derivative of an Implicit Function


The method we use is to differentiate, term by term, w.r.t. x, but we first need to know how to
differentiate terms like 𝑦 3 with respect to x.
If 𝑦 = 𝑓(𝑥)

then
𝑦 3 = [𝑓(𝑥)]3

We can apply the chain rule that we applied in earlier sections by referring to the standard
derivatives formula list

Therefore, by Formula no. 1, we have


𝑑
[𝑓(𝑥)]3 = 𝑛[𝑓(𝑥)]𝑛−1 𝑓 ′ (𝑥) = 3[𝑓(𝑥)]2 𝑓 ′ (𝑥)
𝑑𝑥

Since 𝑦 = 𝑓(𝑥), we therefore have


𝑑 3
𝑦 = 3𝑦 2 𝑦 ′
𝑑𝑥

Likewise when 𝑔(𝑦) = 𝑒 𝑦 , from Fomula no. 2, we have


𝑑 𝑦 𝑑 𝑓(𝑥)
𝑒 = 𝑒 = 𝑒 𝑓(𝑥) 𝑓 ′ (𝑥) = 𝑒 𝑦 𝑦 ′
𝑑𝑥 𝑑𝑥

that is
𝑑 𝑦
𝑒 = 𝑒𝑦𝑦′
𝑑𝑥

We can now differentiate, term by term with respect to x, the example considered above, i.e.,
if
𝑥2 − 𝑦2 + 𝑦 = 1

76
then
(𝑥 2 )′ − (𝑦 2 )′ + (𝑦)′ = (1)′
→ 2𝑥 − 2𝑦𝑦 ′ + 𝑦 ′ = 0
→ −2𝑦𝑦 ′ + 𝑦 ′ = −2𝑥
→ (1 − 2𝑦)𝑦 ′ = −2𝑥
−2𝑥 2𝑥
→ 𝑦′ = =
1 − 2𝑦 2𝑦 − 1
𝑑𝑦
Remember 𝑑𝑥 = 𝑦 ′ and so the above is correct as it is. In fact, I will be using the notation 𝑦 ′
more often now as it is less cumbersome to use in implicit differentiation as some of the
implicit functions can be long. Implicit functions also obey the same product, quotient and
chain rule that we have been working with on explicit differentiation. Let’s explore these
rules in the implicit functions by means of some examples. The examples have been made
deliberately simple so that you focus on one rule at a time but that is an idealisation, in
practice, you will meet all the rules within the same question.

Example
𝑑𝑦
Find 𝑑𝑥 of the following functions:
a) 𝑥𝑦 = 𝑥 + 3
𝑦
b) 𝑥 = 5 + 𝑥 3
c) 𝑥 = sin 𝑦 2

Solution
a)
𝑥𝑦 = 5

Differentiating both sides with respect to x implies


(𝑥𝑦)′ = (5)′
→ 𝑥𝑦 ′ + 𝑦 = 0
→ 𝑥𝑦 ′ = −𝑦
𝑦
∴ 𝑦′ = −
𝑥

b)
𝑦
= 5 + 𝑥3
𝑥

For brevity, we ignore the fact that you can multiply throughout by x. And so differentiating
both sides w.r.t. x implies

𝑦 ′
� � = (5)′ + (𝑥 3 )′
𝑥
𝑥𝑦 ′ − 𝑦 ∙ 1
→ = 0 + 3𝑥 2
𝑥2
→ 𝑥𝑦 ′ − 𝑦 = 3𝑥 4
→ 𝑥𝑦 ′ = 3𝑥 4 + 𝑦

3𝑥 4 + 𝑦
→ 𝑦 =
𝑥

77
c)
𝑥 = sin 𝑦 2

Differentiating throughout with respect to x implies


→ (𝑥)′ = (sin 𝑦 2 )′
→ 1 = cos 𝑦 2 ∙ 2𝑦 ∙ 𝑦 ′
1
→ = 𝑦′
2𝑦 cos 𝑦 2
1
→ 𝑦′ =
2𝑦 cos 𝑦 2

Maybe let’s look at one more example before jumping to the next section.

Example
𝑑𝑦
Find 𝑑𝑥 if
�𝑥 2 + 𝑦 2 = 𝑥 + 𝑦

Solution
I will keep the discussions to a minimum, the solution is self-explanatory, otherwise develop
your own notes here.
�𝑥 2 + 𝑦 2 = 𝑥 + 𝑦
1 ′
→ �(𝑥 2 + 𝑦 2 )2 � = (𝑥)′ + (𝑦)′
1 2 1
→ (𝑥 + 𝑦 2 )−2 ∙ (2𝑥 + 2𝑦𝑦 ′ ) = 1 + 𝑦′
2
𝑥 𝑦𝑦′
→ + = 1 + 𝑦′
�𝑥 2 + 𝑦 2 �𝑥 2 + 𝑦 2
𝑦𝑦′ 𝑥
→ − 𝑦′ = 1 −
�𝑥 2 + 𝑦 2 �𝑥 2 + 𝑦 2
𝑦 𝑥
→ � − 1� 𝑦 ′ = 1 −
�𝑥 2 + 𝑦 2 �𝑥 2 + 𝑦 2
𝑥
1−
�𝑥 + 𝑦 2
2
→ 𝑦′ = 𝑦
−1
�𝑥 2 + 𝑦 2
�𝑥 2 + 𝑦 2 − 𝑥
�𝑥 2 + 𝑦 2
→ 𝑦′ =
𝑦 − �𝑥 2 + 𝑦 2
�𝑥 2 + 𝑦 2
�𝑥 2 + 𝑦 2 − 𝑥
∴ 𝑦′ =
𝑦 − �𝑥 2 + 𝑦 2

NB: Pay close attention to how I simplified 𝑦′.

78
Mistake 18
𝑑𝑦
Find 𝑑𝑥 if
𝑥2 − 𝑦2 + 𝑦 = 1

Typical student answer


2𝑥 − 2𝑦 + 𝑦 ′ = 0
→ 𝑦 ′ = 2𝑦 − 2𝑥
COMMENT: Never forget 𝑦′ after differentiation any y term, like above in the derivative of
𝑦 2 where the 𝑦′ is somehow left out leading to an incorrect solution. You will be penalised if
you leave out 𝒚′ after differentiating any y term and you may lose the rest of the marks
for that question.

Mistake 19
𝑑𝑦
Find 𝑑𝑥 if
𝑥2 − 𝑦2 + 𝑦 = 1

Typical student solution


𝑑𝑦
= 2𝑥 − 2𝑦𝑦 ′ + 𝑦 ′ = 0
𝑑𝑥
𝑑𝑦
Haibo, this statement is meaningless. The only reason why we had 𝑑𝑥 on the left hand side of
𝑑𝑦
earlier solutions is that there was only y on the left hand side and the derivative of y is 𝑑𝑥 . In
𝑑𝑦 𝑑𝑦
this case, you have to solve for 𝑑𝑥 itself so you should not write 𝑑𝑥 as if you are dealing with
an explicit function of x..

To avoid landing yourself in trouble when differentiating, avoid putting equal signs in
the same line, it easily distracts your work. You will be penalised if you repeat the above
statement in a test and/or examination.

11.3 Logarithmic Differentiation


We are now going to examine some equations which are awkward to differentiate as they
stand, but which are much easier to deal with if we first take the logs of both sides of the
equation. One type of equation where this method is useful is one which is of the form
𝑓(𝑥) 𝑔(𝑥) . You cannot apply any of the laws we discussed so far if both the base and the index
are not constants but functions of x.

Example
𝑑𝑦
Find 𝑑𝑥 if
𝑦 = 𝑥𝑥

Solution
𝑦 = 𝑥𝑥
→ ln 𝑦 = ln 𝑥 𝑥
→ ln 𝑦 = 𝑥 ln 𝑥

79
Differentiating both sides with respect to x implies
1 ′ 1
∙ 𝑦 = ln 𝑥 + 𝑥 ∙
𝑦 𝑥
𝑦′
→ = ln 𝑥 + 1
𝑦
→ 𝑦 ′ = 𝑦(ln 𝑥 + 1)

But 𝑦 = 𝑥 𝑥 , therefore
𝑦 ′ = 𝑥 𝑥 (ln 𝑥 + 1)

NB: Always remember to perform the reverse substitution in the final step after performing
logarithmic differentiation. You may be penalised if you don’t perform the reverse
substitution in the final step.

Mistake 20
𝑑𝑦
Find 𝑑𝑥 if
𝑦 = 𝑥 2𝑥 − 𝑥

Typical student answer


𝑦 ′ = 2𝑥 ∙ 𝑥 2𝑥−1 − 1
→ 𝑦 ′ = 2𝑥 2𝑥 − 1

COMMENT: This is a straight zero, on one hand you are treating x as a variable and on the
other you are treating it as a constant. This signals confusion with the principles of
differentiation.

TIP: Always re-arrange your terms so that you have the awkward term on the right hand side
and all the other terms on the left. It is equally pointless to take the logarithm on the question
as it is since you cannot obtain any meaningful simplification.

Solution
𝑦 = 𝑥 2𝑥 − 𝑥
→ 𝑦 + 𝑥 = 𝑥 2𝑥
→ ln(𝑦 + 𝑥) = ln 𝑥 2𝑥
→ ln(𝑦 + 𝑥) = 2𝑥 ln 𝑥
1 1
→ ∙ (𝑦 ′ + 1) = 2 ln 𝑥 + 2𝑥 ∙
𝑦+𝑥 𝑥
𝑦′ 1
→ + = 2 ln 𝑥 + 2
𝑦+𝑥 𝑦+𝑥
𝑦′ 1
→ = 2 ln 𝑥 + 2 −
𝑦+𝑥 𝑦+𝑥
1
→ 𝑦 ′ = (𝑦 + 𝑥) �ln 𝑥 2 + 2 − �
𝑦+𝑥
1
∴ 𝑦 ′ = 𝑥 2𝑥 �ln 𝑥 2 + 2 − 2𝑥 �
𝑥

NB: Brackets always add to the clarity of your work, use them.

80
Mistake 21
𝑑𝑦
Find 𝑑𝑥 if
ln(𝑥 + 𝑦) = 𝑥 + 𝑦

Typical student answer


1
∙ 1 + 𝑦 ′ = 1 + 𝑦′
𝑥+𝑦

NB: You may be penalised if you don’t put the brackets on the left hand side. Leaving
out may in itself change your solution resulting in you getting a zero for that question
even if you have the correct idea about how to differentiate.

Mistake 22
𝑑𝑦
Find 𝑑𝑥 if
𝑦 = 𝑥 2𝑥 − 𝑥

Typical student answer


ln 𝑦 = ln 𝑥 2𝑥 − ln 𝑥

COMMENT: This is a violation of the application of the logarithm and is a straight zero, go
back and revise the laws of logarithms.

Mistake 23
𝑑𝑦
Find 𝑑𝑥 if
sin 𝑥 + cos 𝑥
𝑦=
sin 𝑥 − cos 𝑥

Typical student answer


(sin 𝑥 − cos 𝑥)(cos 𝑥 − sin 𝑥) − (sin 𝑥 + cos 𝑥) (cos 𝑥 + sin 𝑥)
𝑦′ =
(sin 𝑥 − cos 𝑥)2

COMMENT: If the question doesn’t specify that you must use the quotient, you are free to
re-arrange the equation before you differentiate. The quotient rule in most cases creates terms
which are cumbersome to work with leaving you exposed to a high risk of making
unintended errors which cost you in the process. Of course, sometimes it is completely
unavoidable to use the quotient rule hence you must master it.

Suggested solution
You begin by multiplying throughout by sin 𝑥 − cos 𝑥 to clear out the product and
differentiate implicitly giving us
sin 𝑥 + cos 𝑥
𝑦=
sin 𝑥 − cos 𝑥
→ 𝑦(sin 𝑥 − cos 𝑥) = sin 𝑥 + cos 𝑥
→ 𝑦′(sin 𝑥 − cos 𝑥) + 𝑦(cos 𝑥 + sin 𝑥) = cos 𝑥 − sin 𝑥

You can finish up the rest since this is now familiar territory.

81
Turn to page 28 of the study guide. From Tutorial 4, do Question1 and Question 2.
In the next section, we will now be looking at the application of differentiation to determine
the maxima and minima of functions, as well as equations of a normal and a tangent.

REFLECTION AND COMMENT


In the last unit, we introduced the product rule, the quotient rule and the chain rule. So far, we
have been dealing with functions in which we can easily separate the x and y variables. In
other words, all functions have been of the form y = f ( x ) . In this unit, we went to find the
derivate in cases where it is difficult or impossible to separate the variables. For such
functions, the derivative is evaluated implicitly. This means you differentiate term by term
using a combination of the product rule, the chain rule, and/or the quotient rule. For example,
dy
if we have cos ( x + y ) = x 2 + xy 2 , to evaluate , we say
dx
− sin ( x + y ) ⋅ (1 + y ') = 2 x + y 2 + 2 xyy '
dy
and go on to simplify and solve for .
dx
To differentiate cos ( x + y ) you use the chain rule since it is a function of a function.
Differentiating x 2 by now should be elementary whereas to differentiate xy 2 , we use the
product rule since we have a product of two functions, x and y 2 .
To differentiate y 2 , we use the chain rule. In fact, we can generalise to say the derivative of
y n is n ⋅ y n −1 ⋅ y '.
We also looked at some functions for which we need to first take the logarithm before
differentiating. These are functions of the form
g( x)
= y f ( x) + h ( x)
To differentiate the above function, it is necessary to first ‘collect like terms’:
g( x)
y − h ( x) = f ( x)
and then take the natural logarithm on both sides and apply the law of logarithms to get:
ln ( y − h ) = ln f g
⇒ ln ( y − h ) =
g ln f
The derivative of the above alternative expression of the function can now be easily obtained
via the chain rule on the left hand side and the product rule on the right hand side.

At this point, we have covered all the techniques of differentiation for this course (and
beyond). We now move on to the applications of differentiation in the next unit where we get
to explore the application of the derivative in real life.

82
12 Applications of Differentiation
Learning Outcomes
After completing this section, you should be able to
• Find the gradient of a curve at a point
• Find equations of the tangent and the normal
• Find higher order derivatives
• Find the coordinates of the maximum and minimum turning points with verifications
• Given a displacement function, find the velocity and acceleration functions for both
linear and circular motion

12.1 Finding the gradient at a particular point on a curve


𝑑𝑦
At the beginning of this content area, we defined 𝑑𝑥 as a gradient function. To get the
gradient at a particular point therefore, it is a simple matter of substituting the coordinates of
𝑑𝑦
that point into 𝑑𝑥 . In an explicit function, you only need to substitute the x-values whereas in
𝑑𝑦
an implicit function, you need to substitute both the x and y values into 𝑑𝑥 to get m, the
gradient at that point.

12.2 Determining the equations of a tangent and a normal


The line that touches the curve at a particular point is called the tangent at that point. Notice
that the word touch has a precise mathematical meaning. A line that meets a curve and carries
on without crossing to the other side of the curve at that point, is said to touch the curve at the
point of contact.

The gradient of the curve at that point of contact is the same as gradient of the tangent to the
curve at that point.

The normal at a point is perpendicular to the tangent at that point. If 𝑚 𝑇 is the gradient of the
tangent at a point and 𝑚𝑁 is the gradient of the normal at that point, we have

𝑚 𝑇 × 𝑚𝑁 = −1

I am sure you have come across the above relationship before.


The equation is a straight line is given by

𝑦 = 𝑚𝑥 + 𝑐

or
𝑦 − 𝑦1 = 𝑚(𝑥 − 𝑥1 )
𝑑𝑦
Therefore, by evaluating 𝑑𝑥 at a particular point, we obtain 𝑚 𝑇 and if we want to find the
1
gradient of the normal, we simply say 𝑚𝑁 = − 𝑚 . Substituting 𝑚 𝑇 or 𝑚𝑁 into the equation
𝑇
of a straight line gives us the equation of a tangent or a normal, respectively.

Example
Find the equation of the tangent and the equation of the normal to the curve

83
3𝑥 3 𝑦 2 − 2𝑦 3 = −4 at the point (1;2).

Solution
3𝑥 3 𝑦 2 − 2𝑦 3 = −4
→ 9𝑥 2 𝑦 2 + 6𝑥 3 𝑦𝑦 ′ − 6𝑦 2 𝑦 ′ = 0
→ 𝑦 ′ (6𝑥 3 𝑦 − 6𝑦 2 ) = −9𝑥 2 𝑦 2

9𝑥 2 𝑦 2 3𝑥 2 𝑦
→ 𝑦′ = − 3 =
6𝑥 𝑦 − 6𝑦 2 2(𝑦 − 𝑥 3 )

The gradient of the tangent at (1;2) is given by


3(1)2 (2) 6
𝑦 ′ |(1;2) = = =3
2(2 − 13 ) 2

From the equation of a straight line, we get the equation of the tangent as follows
𝑦 = 𝑚𝑇 𝑥 + 𝑥
→ 2= 3×1+𝑐
→ 𝑐 = −1
∴ 𝑦 = 3𝑥 − 1

The gradient of the normal is given by


1
𝑚𝑁 = − 1�𝑚 𝑇 = −
3

Therefore, the equation of the normal is given by


𝑦 − 𝑦1 = 𝑚(𝑥 − 𝑥1 )
1
→ 𝑦 − 2 = − (𝑥 − 1)
3
𝑥
→ 𝑦−2=− +1
3
𝑥
𝑦 =− +3
3

NB: The use of different equations of a straight line was for illustrative purposes only;
choose the approach you are most comfortable with.

Turn to page 29 of the study guide and from Question 3, do questions 3.1 and 3.2
only.

12.3 Higher Order Derivatives


Consider the function
𝑦 = 4𝑥 3 + 3𝑥 2 − 6𝑥 − 1

Differentiating throughout w.r.t. x, we get


𝑦 ′ = 12𝑥 2 + 6𝑥 − 6
𝑑𝑦
𝑦′ or 𝑑𝑥 is called the first derivative. If we differentiate 𝑦′, we get the second derivative

84
𝑦 ′′ = 12𝑥 + 6

An alternative notation for 𝑦′′ is


𝑑2 𝑦
𝑑𝑥 2

Still, we can differentiate 𝑦′′ to get


𝑦 ′′′ = 12

𝑑3 𝑦
𝑦 ′′′ or 𝑑𝑥 3
is called the third derivative. In general, the formula for the nth derivative is given
by
𝑑𝑛 𝑦
𝑦𝑛 =
𝑑𝑥 𝑛

where n in 𝑦 𝑛 is given in Roman numerals to avoid confusion with the notation for the nth
power of y.

12.4 Stationary Values


A stationary point is a point where 𝑓 ′ (𝑥) = 0. At a stationary point, we say y or f(x) has a
stationary value and the tangent to the curve y=f(x) is parallel to the x-axis.
A turning point is a point where the curves ‘turns’ literally. There are two types of turning
points, the maximum (plural maxima) and the minimum (plural minima). The terms
maximum and minimum apply only to the behaviour of the curve in the neighbourhood of a
stationary point and does not necessarily mean the greatest/least value of y overall.

𝑑𝑦
To locate a stationary point, we evaluate 𝑑𝑥 = 0 and get the x values from which we
substitute back into the original function to the get y value at the stationary point.
To determine the nature of a stationary point, that is whether it is maximum or minimum, we
use the second derivative test.

𝑑𝑦 𝑑2 𝑦
• At a maximum point, 𝑑𝑥 = 0 and 𝑑𝑥 2 < 0
𝑑𝑦 𝑑2 𝑦
• At a minimum point, 𝑑𝑥 = 0 and 𝑑𝑥 2 > 0

NB: You will get zero if you don’t do the second derivative test to determine the nature
of the stationary points so understand the points above.

Example
Find the stationary points on the following curve and distinguish between them.
𝑥3
𝑦 = 4𝑥 −
3

Solution
𝑥3
𝑦 = 4𝑥 −
3
𝑑𝑦
→ = 4 − 𝑥2
𝑑𝑥

85
𝑑2𝑦
→ = −2𝑥
𝑑𝑥 2
𝑑𝑦
At a stationary point, 𝑑𝑥 = 0, therefore
→ 4 − 𝑥2 = 0
→ (2 − 𝑥)(2 + 𝑥) = 0
→ 𝑥 = 2 or 𝑥 = −2

23 8 16
When 𝑥 = 2, 𝑦 = 4(2) − = 8−3=
3 3
(−2)3 8 16
When 𝑥 = −2, 𝑦 = 4(−2) − = −8 + 3 = −
3 3

16
Now, at �2; 3 �
𝑑2𝑦
= −2(2) = −4 < 0
𝑑𝑥 2
16
Therefore, �2; 3 � is a maximum point.

16
At �−2; − 3 �
𝑑2𝑦
= −2(−2) = 4 > 0
𝑑𝑥 2
16
Therefore, �−2; − 3 � is a minimum point.

NB: Do not confuse the minimum and the maximum point, many students do this mistake.
You will be penalised for pointing at the wrong maximum or minimum.

Mistake 24
𝑑2 𝑦
Calculating 𝑑𝑥 2 = 0 to determine a stationary point.
𝑑𝑦
This is a straight zero, to determine a stationary point, use 𝑑𝑥 = 0.

Turn to page 30 of the study guide and do question 3.3.

Mistake 25
𝑑𝑦
Find 𝑑𝑥 if
𝑥2𝑦3 = 𝑥 − 𝑦

Typical student answer


→ 6𝑥𝑦 2 𝑦 ′ = 1 − 𝑦′

COMMENT: This is a straight zero because the left hand side is clearly a product and the
rules of differentiation are clearly being violated.

Otherwise, the solution should proceed as follows:

86
𝑥2𝑦3 = 𝑥 − 𝑦
→ 2𝑥𝑦 3 + 3𝑥 2 𝑦 2 𝑦 ′ = 1 − 𝑦′

12.5 Displacement, velocity, acceleration: Linear Motion


Displacement is defined as the distance moved in a stated direction. The unit of measurement
is metres, 𝑚.

Velocity is defined as the rate of change of displacement or the change of displacement per
unit time. The unit of measurement is metres per second, 𝑚⁄𝑠 or 𝑚𝑠 −1 . Using calculus, it is
the first derivative of the displacement.

Acceleration is defined as the rate of change of velocity or the change in velocity per unit
time. The unit of measurement is metres per second squared, 𝑚⁄𝑠 2 or 𝑚𝑠 −2. Using calculus,
it is defined as the derivative of velocity or the second derivative of displacement.

For motion in a straight line, displacement has the symbol 𝑠 and since it’s a function time, we
have 𝑠(𝑡).

The symbol for velocity is 𝑣 = 𝑣(𝑡) and for acceleration it is 𝑎 = 𝑎(𝑡).

From the definitions, we have


𝑑𝑠
𝑣=
𝑑𝑡
𝑑𝑣 𝑑2 𝑠
𝑎= =
𝑑𝑡 𝑑𝑡 2
NB: Never leave out the units.

Turn to page 30 of the study guide. From Tutorial 5, on Question 1, do questions


1.1 to 1.3.2.

12.6 Displacement, velocity and acceleration: Circular Motion


The definitions above for displacement, velocity and acceleration also hold for circular
motion. However, the difference is in the units used since in circular motion, we are more
concerned with the angle turned and so the terms change to angular displacement, angular
velocity and angular acceleration.

For circular motion, the symbol for angular displacement is 𝜃 = 𝜃(𝑡) and the unit of
measurement is the radian.

Angular velocity is defined as the rate of change of angular displacement and the symbol for
angular velocity is 𝜃̇. The unit of measurement is the radian per second, rad⁄𝑠 or rad 𝑠 −1.
Angular acceleration is defined as the rate of change of angular velocity and the symbol for
angular acceleration is 𝜃̈. The unit of measurement is the radian per second squared, rad⁄𝑠 2
or rad 𝑠 −2.

87
From the definitions of angular velocity and angular acceleration, we have
𝑑𝜃
𝜃̇ =
𝑑𝑡
𝑑2 𝜃
𝜃̈ = 2
𝑑𝑡
NB: Always remember the units.

Turn to page 31 of the study guide and do questions 1.4 to 1.4.2


At this point, you have covered enough material to take you through Semester Test 2. Make
sure you do a thorough revision and aim high for your DP.

REFLECTION AND COMMENT


The value of a derivative of a slope at a particular point of the slope equals the value of the
gradient of a tangent at that point. If mT is the value of the gradient of a tangent to the slope
and mN is the value of a normal to the slope, then mT × mN = −1. The point at which the
gradient of a slope is zero is called the stationary point or the turning point.
The turning point is a maximum if
dy
=0
dx
AND
d2y
< 0.
dx 2

On the other hand, the turning point is a minimum if


dy
=0
dx
AND
d2y
>0
dx 2
For both linear and circular motion,
velocity is the first derivative of the displacement function with respect to time, and
acceleration is the second derivative of displacement with respect to time.

88
CONTENT AREA 4: INTEGRAL CALCULUS

INTEGRAL CALCULUS

Integration as anti-differentiation

Standard Integrals

Indefinite integrals

Area under the Curve

89
13 Basic Integration
Learning Outcomes
After completing this section, you should be able to
• Appreciate that integration is the reverse process of differentiation
• Recognise the need for a constant of integration
• Evaluate the indefinite integrals of standard forms
• Evaluate the indefinite integrals of polynomials
• Evaluate the indefinite functions of composite functions
• Integrate all algebraic functions, transcendental and trigonometric functions

13.1 Differentiation Revisited


When 𝑥 3 is differentiated with respect to x, the derivative is 3𝑥 2 .
Conversely, if the derivative of an unknown function is 3𝑥 2 then it is clear that the unknown
function could be 𝑥 3 .

This process of finding a function from its derivative, which reverses the operation of
differentiating, is called integrating.
The Constant of Integration
As seen above, 3𝑥 2 is the derivative of 𝑥 3 , but it is also the derivative of 𝑥 3 + 5, 𝑥 3 − 7, and
in fact, the derivative of any 𝑥 3 + any constant.
Therefore the result of integrating 3𝑥 2 , which is called the integral of 3𝑥 2 , is not a unique
function but it is of the form
𝑥3 + 𝐶

where 𝐶 is any constant. Note: any letter can be used to represent a constant.
This is written
� 3𝑥 2 𝑑𝑥 = 𝑥 3 + 𝐶

where ∫ ⋯ 𝑑𝑥 means the integral of ⋯ w.r.t. x.


Integrating any function reverses the process of differentiating so, for any function f(x), we
have

𝑑
� 𝑓(𝑥) 𝑑𝑥 = 𝑓(𝑥) + 𝐶
𝑑𝑥

Since as we have noted above,


� 3𝑥 2 𝑑𝑥 = 𝑥 3 + 𝐶

it follows that
𝑥3
� 𝑥 2 𝑑𝑥 = +𝐶
3
1
C represents any constant so there is no need to re-write it as 3 𝐶.

90
In general, the derivative of 𝑥 𝑛+1 is (𝑥 + 1)𝑥 𝑛 therefore
1
� 𝑥 𝑛 𝑑𝑥 = 𝑥 𝑛+1 + 𝐶
(𝑛 + 1)
where 𝑛 ≠ −1. The case for 𝑛 = −1 is deal with later in this content area.

13.2 Integrating a Sum or Difference of Functions


We have already seen that to differentiate a sum or difference of functions, we differentiate
the function term by term. Likewise for integration, which is the reverse for differentiation,
integration of a sum or difference of functions is also done term by term.

Example
1
Find the integral of 𝑥 3 + √𝑥 + .
√𝑥

Solution
1 1 1
� �𝑥 3 + √𝑥 + � 𝑑𝑥 = � �𝑥 3 + 𝑥 2 + 𝑥 −2 � 𝑑𝑥
√𝑥
3 1
𝑥4 𝑥2 𝑥2
= + + +𝐶
4 �3� �1�
2 2
3
4
𝑥 2𝑥 2 1
= + + 2𝑥 2 + 𝐶
4 3

Integrate the following with respect to x


a) 𝑥 5
4
b) √𝑥
1
c) 5
𝑥2
d) 𝑥 −3

13.3 Integrating (𝒂𝒙 + 𝒃)𝒏


Let us start by considering the function 𝑓(𝑥) = (2 + 3𝑥)6 .
To differentiate f(x), we refer to Formula no. 1 (page 39 study guide) to get
𝑑
(2 + 3𝑥)6 = 𝑛[𝑓(𝑥)]𝑛−1 𝑓 ′ (𝑥) = 6(2 + 3𝑥)5 (3) = (6)(3)(2 + 3𝑥)5
𝑑𝑥

Hence
�(6)(3)(2 + 3𝑥)5 𝑑𝑥 = (2 + 3𝑥)6 + 𝐶
or
1
�(2 + 3𝑥)5 𝑑𝑥 = (2 + 3𝑥)6 + 𝐶
(3)(6)

Considering 𝑓(𝑥) = (𝑎𝑥 + 𝑏)𝑛+1 in a similar way gives the general result
1
�(𝑎𝑥 + 𝑏)𝑛 𝑑𝑥 = (𝑎𝑥 + 𝑏)𝑛+1 + 𝐶
(𝑎)(𝑛 + 1)

91
An alternative way to evaluate ∫(𝑎𝑥 + 𝑏)𝑛 𝑑𝑥 will be to use substitution as follows
Let
𝑢 = 𝑎𝑥 + 𝑏
Therefore
𝑑𝑢
=𝑎
𝑑𝑥
𝑑𝑢
If we treat 𝑑𝑥 as a fraction, we can cross-multiply to get
𝑑𝑢
= 𝑑𝑥
𝑎

Now, substituting for 𝑑𝑥 and 𝑢 in the integral ∫(𝑎𝑥 + 𝑏)𝑛 𝑑𝑥 gives


𝑑𝑢 1
�(𝑎𝑥 + 𝑏)𝑛 𝑑𝑥 = � 𝑢𝑛 ∙ � � = � 𝑢𝑛 𝑑𝑢
𝑎 𝑎

You can see that we have changed the variables of integration from x to u and just like for
differentiation, the integration of the variables with respect to u follows the same rules as the
integration with respect to x that we are used to.
Therefore
𝑛
1 𝑛
1 𝑢𝑛+1 1
�(𝑎𝑥 + 𝑏) 𝑑𝑥 = � 𝑢 𝑑𝑢 = +𝐶 = 𝑢𝑛+1 + 𝐶
𝑎 (𝑛
𝑎 + 1) (𝑎)(𝑛 + 1)

However, since 𝑢 = 𝑎𝑥 + 𝑏, we do the reverse substitution for u to finally get


1
�(𝑎𝑥 + 𝑏)𝑛 𝑑𝑥 = (𝑎𝑥 + 𝑏)𝑛+1 + 𝐶
(𝑎)(𝑛 + 1)

I find the substitution method quite straight forward and really handy and will be using it
everywhere necessary in the rest of this section.

NB: The importance of writing 𝑑𝑥 in the integral expression should apparent from the
substitution. The 𝑑𝑥 term multiplies each term under the integral sign hence the function to
be integrated is put into brackets. You may be penalised if you leave out the 𝒅𝒙 term from
an integral expression.

Example
1
Integrate (4𝑥 − 1)3 with respect to x.

Solution
Let 𝑢 = 4𝑥 − 1, therefore
𝑑𝑢
=4
𝑑𝑥
𝑑𝑢
→ = 𝑑𝑥
4
Therefore
1 1 𝑑𝑢 1 1
�(4𝑥 − 1)3 𝑑𝑥 = � 𝑢3 ∙ � � = � 𝑢3 𝑑𝑢
4 4

92
4
1 𝑢3 3 4
= � �+𝐶 = 𝑢3 + 𝐶
4 4 16
3

By reverse substitution therefore,


1 3 4
�(4𝑥 − 1)3 𝑑𝑥 = (4𝑥 − 1)3 + 𝐶
16

Determine the following integrals by using appropriate substitution


a) ∫(2 − 5𝑥)2 𝑑𝑥
b) ∫(4 − 3𝑥)−5 𝑑𝑥
c) ∫ √2 − 3𝑥 𝑑𝑥
1
d) ∫ 1−𝑥 𝑑𝑥
e) ∫ √3𝑥 + 7 𝑑𝑥
1
f) ∫ �𝑥 3 − (1−𝑥)3 � 𝑑𝑥
g)

13.4 Standard Integrals


Whenever a function f(x) is recognised as the derivative of a function f(x) then
𝑑
𝑓(𝑥) = 𝑓 ′ (𝑥)𝑑𝑥 ⇒ � 𝑓 ′ (𝑥) 𝑑𝑥 = 𝑓(𝑥) + 𝐶
𝑑𝑥
Thus an integral whose derivative is known can be established as a standard integral.

13.5 Integrating Exponential Functions


𝑑
From your lectures, you already know that 𝑑𝑥 𝑒 𝑥 = 𝑒 𝑥
hence
� 𝑒 𝑥 𝑑𝑥 = 𝑒 𝑥 + 𝐶

It also follows that


� 𝑐𝑒 𝑥 𝑑𝑥 = 𝑐 � 𝑒 𝑥 𝑑𝑥 = 𝑐𝑒 𝑥 + 𝐶

We can use the standard formula for ∫ 𝑒 𝑥 𝑑𝑥 and substitution to generate the integral of
∫ 𝑒 𝑎𝑥+𝑏 𝑑𝑥 as follows.

Example
Evaluate ∫ 𝑒 𝑎𝑥+𝑏 𝑑𝑥

Solution
𝑑𝑢
Let 𝑢 = 𝑎𝑥 + 𝑏 ⇒ 𝑑𝑥
=𝑎
𝑑𝑢
∴ = 𝑑𝑥
𝑎

93
Hence
𝑑𝑢 1
� 𝑒 𝑎𝑥+𝑏 𝑑𝑥 = � 𝑒 𝑢 � � = � 𝑒 𝑢 𝑑𝑢
𝑎 𝑎
1 𝑢
= 𝑒 +𝐶
𝑎
By reverse substitution therefore, we have
1 𝑎𝑥+𝑏
� 𝑒 𝑎𝑥+𝑏 𝑑𝑥 = 𝑒 +𝐶
𝑎

Example
Evaluate ∫ 2𝑒 1−5𝑥 𝑑𝑥

Solution
𝑑𝑢
Let 𝑢 = 1 − 5𝑥 ⇒ 𝑑𝑥
= −5
𝑑𝑢
∴ − = 𝑑𝑥
5

Hence the integral becomes


𝑑𝑢
� 2𝑒 1−5𝑥 𝑑𝑥 = 2 � 𝑒 1−5𝑥 𝑑𝑥 = 2 � 𝑒 𝑢 �− �
5
2 2
= − � 𝑒 𝑢 𝑑𝑢 = − 𝑒 𝑢 + 𝐶
5 5
2
∴ � 2𝑒 1−5𝑥 𝑑𝑥 = − 𝑒 1−5𝑥 + 𝐶
5

Integrate each function with respect to x


a) 𝑒 4𝑥
b) 4𝑒 −𝑥
c) 𝑒 3𝑥−2
d) 6𝑒 −2𝑥
e) 5𝑒 𝑥−3
f) 𝑒 (2+𝑥⁄2)

You already know from differentiation Formula no. 3 that


𝑑 𝑥
𝑎 = 𝑎 𝑓(𝑥) 𝑓 ′ (𝑥) ln 𝑎 = 𝑎 𝑥 ∙ 1 ∙ ln 𝑎 = 𝑎 𝑥 ln 𝑎
𝑑𝑥

Therefore
�(𝑎 𝑥 ) ln 𝑎 𝑑𝑥 = ln 𝑎 � 𝑎 𝑥 𝑑𝑥 = 𝑎 𝑥 + 𝐶
1 𝑥
⇒ � 𝑎 𝑥 𝑑𝑥 = 𝑎 +𝐶
ln 𝑎

We can use the above standard integral to integrate functions of the form 𝑎𝑏𝑥+𝑐

Example
Integrate 2(3𝑥−2) with respect to x.

94
Solution
There are two equivalent approaches in this type of integral and I will do them both. You are
free to select the one you are confident with.

• Approach (1)
𝑑𝑢
Let 𝑢 = 3𝑥 − 2 ⇒ 𝑑𝑥
=3
𝑑𝑢
∴ = 𝑑𝑥
3

Therefore
𝑑𝑢 1 1 1 𝑢
� 2(3𝑥−2) 𝑑𝑥 = � 2𝑢 � � = � 2𝑢 𝑑𝑢 = � 2 �+𝐶
3 3 3 ln 2
1 1 (3𝑥−2)
∴ � 2(3𝑥−2) 𝑑𝑥 = 2(3𝑥−2) + 𝐶 = 2 +𝐶
3 ln 2 ln 8

• Approach (2)
23𝑥 1
� 2(3𝑥−2) 𝑑𝑥 = � 2
𝑑𝑥 = � 23𝑥 𝑑𝑥
2 4
𝑑𝑢
Let 𝑢 = 3𝑥 ⇒ 𝑑𝑥 = 3
𝑑𝑢
= 𝑑𝑥
3
Hence
1 1 𝑑𝑢 1
� 23𝑥 𝑑𝑥 = � 2𝑢 � � = � 2𝑢 𝑑𝑢
4 4 3 12
1 1 𝑢 1
= � 2 �+𝐶 = 23𝑥 + 𝐶
12 ln 2 12 ln 2

It is not necessary to go beyond this stage since this is a complete answer but I will do so only
to show that the two solutions are behind.
1
∴ � 2(3𝑥−2) 𝑑𝑥 = 23𝑥 + 𝐶
4 × 3 ln 2
1 1 3𝑥−2
= 2 23𝑥 + 𝐶 = 2 +𝐶
2 × ln 8 ln 8

You are free to choose either of the two alternatives, both solutions are acceptable.

Integrate each function with respect to x


a) 2𝑥
b) 4(2+𝑥)
c) 𝑎1−2𝑥
d) 2𝑥 + 𝑥 2
𝟏
13.6 How to Integrate
𝒙
1
You will have realised that it not possible to use the standard integral for 𝑥 𝑛 to integrate 𝑥.
Check

95
1 𝑥0
� 𝑑𝑥 = +𝐶
𝑥 0

which is undefined because of the division by zero. However, you do remember from
Formula no. 4 that
𝑑 1
ln 𝑥 =
𝑑𝑥 𝑥

Since we know that ln x is defined when x > 0, it therefore implies that


𝑑 1 1
ln 𝑥 = ⇔ � 𝑑𝑥 = ln |𝑥| + 𝐶
𝑑𝑥 𝑥 𝑥

I will explain the use of |x| when we come to definite integration. You need to carefully study
this result as it is of great importance when you get to Mathematics 2 in integration by partial
fractions and homogenous ordinary differential equations.
1
As before, we can easily use the above result to obtain the standard integral for 𝑎𝑥+𝑏

Example
5
Evaluate ∫ 6−7𝑥 𝑑𝑥

Solution
𝑑𝑢
Let 𝑢 = 6 − 7𝑥 ⇒ 𝑑𝑥
= −7
𝑑𝑢
∴ − = 𝑑𝑥
7
Hence
5 5 𝑑𝑢 5 1 5
� 𝑑𝑥 = � �− � = − � 𝑑𝑢 = − ln|𝑢| + 𝐶
6 − 7𝑥 𝑢 7 7 𝑢 7
5 5
∴ � 𝑑𝑥 = − ln|6 − 7𝑥| + 𝐶
6 − 7𝑥 7
5
or 𝐶 = − 7 ln 𝐴
5 5 5 5 𝐴
= − ln|6 − 7𝑥| − ln 𝐴 = − ln 𝐴|6 − 7𝑥| = ln
7 7 7 7 |6 − 7𝑥|

Integrate w.r.t. x giving each answer in a form which


(a) uses C (b) uses ln 𝐴 and is simplified
1
1) 2𝑥
1
2) 3𝑥+1
6
3) 2+3𝑥
3
4) 4−2𝑥
4
5) 1−𝑥

96
13.7 Integrating Trigonometric Functions
The integration of trig functions follows a similar suit. I will illustrate a couple of examples
since the workings are the same as those for the other functions outlined earlier.

Example
Integrate the following functions w.r.t. x
𝜋
a) sin �4 + 𝑥�
𝜋
b) 3 cos �4𝑥 − 2 �
𝜋
c) sec 2 �3 + 2𝑥�

Solution
a)
𝑑
cos 𝑥 = − sin 𝑥 ⇔ � sin 𝑥 𝑑𝑥 = − cos 𝑥 + 𝐶
𝑑𝑥
𝜋 𝑑𝑢
Let 𝑢 = 4 + 𝑥 ⇒ 𝑑𝑥 = 1
∴ 𝑑𝑢 = 𝑑𝑥

Hence
𝜋
� sin � + 𝑥� 𝑑𝑥 = � sin 𝑢 𝑑𝑢 = − cos 𝑢 + 𝐶
4
𝜋 𝜋
∴ � sin � + 𝑥� 𝑑𝑥 = − cos � + 𝑥� + 𝐶
4 4

b)
𝑑
sin 𝑥 = cos 𝑥 ⇔ � cos 𝑥 𝑑𝑥 = sin 𝑥 + 𝐶
𝑑𝑥
𝜋 𝑑𝑢
Let 𝑢 = 4𝑥 − 2 ⇒ 𝑑𝑥 = 4
𝑑𝑢
∴ = 𝑑𝑥
4

Hence
𝜋 𝑑𝑢 3 3
� 3 cos �4𝑥 − � 𝑑𝑥 = � 3 cos 𝑢 � � = � cos 𝑢 𝑑𝑢 = − sin 𝑢 + 𝐶
2 4 4 4
𝜋 3 𝜋
� 3 cos �4𝑥 − � 𝑑𝑥 = − sin �4𝑥 − � + 𝐶
2 4 2
c)
𝑑
tan 𝑥 = sec 2 𝑥 ⇔ � sec 2 𝑥 𝑑𝑥 = tan 𝑥 + 𝐶
𝑑𝑥
𝜋 𝑑𝑢
Let 𝑢 = 3 + 2𝑥 ⇒ 𝑑𝑥 = 2
𝑑𝑢
∴ = 𝑑𝑥
2
Hence
𝜋 𝑑𝑢 1 1
� sec 2 � + 2𝑥� 𝑑𝑥 = � sec 2 𝑢 � � = � sec 2 𝑢 𝑑𝑢 = tan 𝑢 + 𝐶
3 2 2 2
𝜋 1 𝜋
∴ � sec 2 � + 2𝑥� 𝑑𝑥 = tan � + 2𝑥� + 𝐶
3 2 3

97
13.8 Integration Functions of the Form 𝒇′(𝒙)[𝒇(𝒙)]𝒏
The substitution method comes in handy when we want to integrate functions of the form
∫ 𝑓 ′ (𝑥) [𝑓(𝑥)]𝑛 𝑑𝑥. Let us briefly look at a few examples since all the methods we have been
using readily apply in such instances. What counts however is the realisation aspect that
indeed the function is in this form.

Example
Evaluate the following integrals
a) 𝑥√1 − 𝑥 2 𝑑𝑥
b) ∫ cos 2𝑥 + 3 (sin 2𝑥 + 3)2 𝑑𝑥
c) ∫ 𝑒 𝑥 √1 + 𝑒 𝑥 𝑑𝑥
d) ∫ sin 𝜃 �(1 − cos 𝜃)𝑑𝜃
e) ∫ sec 2 𝑥tan3 𝑥 𝑑𝑥
f) ∫(𝑥 + 1) √𝑥 2 + 2𝑥 + 3𝑑𝑥
ln 𝑥
g) ∫ 𝑥 𝑑𝑥
h)
Solution
𝑑𝑢
a) Let 𝑢 = 1 − 𝑥 2 ⇒ 𝑑𝑥 = −2𝑥
𝑑𝑢
∴ = 𝑑𝑥
−2𝑥
Hence
3
𝑑𝑢 1 1 𝑢2
� 𝑥 �1 − 𝑥 2 𝑑𝑥
= � 𝑥√𝑢 �− � = − � √𝑢 𝑑𝑢 = − +𝐶
2𝑥 2 2 �3�
2
1 3
∴ � 𝑥�1 − 𝑥 2 𝑑𝑥 = − (1 − 𝑥 2 )2 + 𝐶
3
𝑑𝑢
b) Let 𝑢 = sin 2𝑥 + 3 ⇒ 𝑑𝑥
= 2 cos(2𝑥 + 3)
𝑑𝑢
= 𝑑𝑥
2 cos(2𝑥 + 3)
Hence
𝑑𝑢 1
� cos(2𝑥 + 3) (sin 2𝑥 + 3)2 𝑑𝑥 = � cos(2𝑥 + 3) 𝑢2 � � = � 𝑢2 𝑑𝑢
2 cos 2𝑥 + 3 2
1 𝑢3 1
� � + 𝐶 = (sin 2𝑥 + 3)3 + 𝐶
=
2 3 6
1
∴ � cos(2𝑥 + 3) (sin 2𝑥 + 3)2 𝑑𝑥 = (sin 2𝑥 + 3)3 + 𝐶
6
𝑑𝑢
c) Let 𝑢 = 1 + 𝑒 𝑥 ⇒ 𝑑𝑥
= 𝑒𝑥
𝑑𝑢
= 𝑑𝑥
𝑒𝑥
Hence
3
𝑑𝑢 𝑢2 2 3
� 𝑒 𝑥 √1 + 𝑒 𝑥 𝑑𝑥 = � 𝑒 𝑥 √𝑢 � 𝑥 � = � √𝑢 𝑑𝑢 = + 𝐶 = 𝑢2 + 𝐶
𝑒 3 3
�2�

98
2 3
∴ � 𝑒 𝑥 √1 − 𝑒 𝑥 𝑑𝑥 = (1 + 𝑒 𝑥 )2 + 𝐶
3
NB: Remember C, the constant of integration
𝑑𝑢
d) Let 𝑢 = 1 − cos 𝜃 ⇒ 𝑑𝜃
= sin 𝜃
𝑑𝑢
∴ = 𝑑𝜃
sin 𝜃
Hence
3
𝑑𝑢 𝑢2
� sin 𝜃√1 − cos 𝜃 𝑑𝜃 = � sin 𝜃 √𝑢 � � = � √𝑢 𝑑𝑢 = +𝐶
sin 𝜃 3
�2�
2 3 2 3
= 𝑢2 + 𝐶 = (1 − cos 𝜃)2 + 𝐶
3 3

NB: Always remember the constant of integration


e) Remember tan3 𝑥 = (tan 𝑥)3
𝑑𝑢
Let 𝑢 = tan 𝑥 ⇒ 𝑑𝑥 = sec 2 𝑥
𝑑𝑢
∴ = 𝑑𝑥
sec 2 𝑥

Hence
𝑑𝑢 𝑢4 (tan 𝑥)4
� sec 2 𝑥 tan3 𝑥 𝑑𝑥 = � sec 2 𝑥 (𝑢3 ) � 2 � = � 𝑢3 𝑑𝑥 = +𝐶 = +𝐶
sec 𝑥 4 4
𝑑𝑢
f) Let 𝑢 = 𝑥 2 + 2𝑥 + 3 ⇒ 𝑑𝑥 = 2𝑥 + 2 = 2(𝑥 + 1)
𝑑𝑢
∴ = 𝑑𝑥
2(𝑥 + 1)
3
𝑑𝑢 1 1 𝑢2
�(𝑥 + 1)�𝑥 + 2𝑥 + 3 𝑑𝑥 = �(𝑥 + 1)√𝑢 �
2 � = � √𝑢 𝑑𝑥 = +𝐶
2(𝑥 + 1) 2 2 �3�
2
3
2
(𝑥 + 2𝑥 + 3)2
= +𝐶
3
1
g) Initially, this looks like a fraction but once we recognise that it is the product of 𝑥 and
𝑑
ln 𝑥, it is clear that 𝑑𝑥 (ln 𝑥) and we can make the substitution 𝑢 = ln 𝑥.
h)
𝑑𝑢 1
Therefore, let 𝑢 = ln 𝑥 ⇒ 𝑑𝑥
=𝑥
𝑥𝑑𝑢 = 𝑑𝑥
Hence
ln 𝑥 𝑢 𝑢2 (ln 𝑥)2
� 𝑑𝑥 = � (𝑥𝑑𝑢) = � 𝑢 𝑑𝑢 = +𝐶 = +𝐶
𝑥 𝑥 2 2
NB: Always remember the constant of integration

99
Turn to page 31 of the study guide. On Question 2, do all the questions from 2.1 to
2.10.

REFLECTION AND COMMENT


In this unit, we introduced integration simply as a reverse process of integration. The
derivative of any constant is zero and therefore all indefinite integrals have a constant of
integration added to them. In most cases, it is necessary to first perform a change of variables
before integrating to simplify your work.

In the next section, we will be looking at the evaluation of definite integrals where limits of
integration are applied.

100
14 Definite Integrals
Learning Outcomes
After completing this section, you should be able to
• Evaluate the definite integrals of standard forms
• Apply the definite integral to find areas between a curve y = f(x), the x-axis and the
vertical lines x = a and x = b.

14.1 Definite Integration


So far, we have been dealing with indefinite integrals in which we get an integral plus a
constant. When limits of integration are applied the integral, we now have what are called
definite integrals. The process of integration is the same as we covered in the previous section
so it is sufficient to start by demonstrating a couple of examples.

𝑏
A definite integral is an integral of the form ∫𝑎 𝑓(𝑥)𝑑𝑥 = [𝐹(𝑥)]𝑏𝑎 = 𝐹(𝑏) − 𝐹(𝑎)

where ∫ 𝑓(𝑥)𝑑𝑥 = 𝐹(𝑥) + 𝐶 a and b are the limits of integration; b is called the upper limit
and a is called the lower limit.

Example
Evaluate each of the following definite integrals
2
a) ∫0 𝑥 3 𝑑𝑥
4
b) ∫2 (𝑥 2 + 4) 𝑑𝑥

Solution
a)
2 2
𝑥4
� 𝑥 𝑑𝑥 = � � = (24 + 𝐶) − (04 + 𝐶) = 16
3
0 4 0

As you can see, the constant of integration cancels out when evaluating definite integrals.
Therefore, the constant of integration is completely omitted during the evaluation of definite
integrals.

b)
4 4
𝑥3 43 23
� (𝑥 2 + 4) 𝑑𝑥 = � + 4𝑥� = � + 4 ∙ 4� − � + 4 ∙ 2�
2 3 2
3 3
64 8 80
= � + 16� − � + 8� =
3 3 3

NB: Leave your final answer as an improper fraction. You should utilise your Casio
calculator in the second last step to obtain the answer straight away.

When there’s a change of variables during integration, it is convenient in such a case to also
change the limits of integration into the new variables. Whereas originally you may have the
limits x = a and x = b, changing the variable from x to u implies the limits also change from u
= c and u = d to avoid having to perform a reverse substitution.

101
Exercise
Evaluate
2
a) ∫1 𝑥 2 (𝑥 3 − 1)4 𝑑𝑥
𝜋⁄4
b) ∫0 sec 2 𝑥𝑒 tan 𝑥 𝑑𝑥

Solution
𝑑𝑢
a) Let 𝑢 = 𝑥 3 − 1 ⇒ 𝑑𝑥
= 3𝑥 2
𝑑𝑢
∴ = 𝑑𝑥
3𝑥 2

When 𝑥 = 2, 𝑢 = 23 − 1 = 8 − 1 = 7
When 𝑥 = 1, 𝑢 = 13 − 1 = 1 − 1 = 0

Hence
2 7
2 (𝑥 3 4
𝑑𝑢 1 7 4 2
� 𝑥 − 1) 𝑑𝑥 = � 𝑥 � 2 � = � 𝑢 𝑑𝑢 (𝑢4 )
1 0 3𝑥 3 0
5 7
1 𝑢 1 57 1 5 16807
= � � = [𝑢 ]0 = (7 − 0) =
3 5 1 15 15 15

𝑑𝑢
b) Let 𝑢 = tan 𝑥 ⇒ 𝑑𝑥
= sec 2 𝑥
𝑑𝑢
∴ = 𝑑𝑥
sec 2 𝑥
When 𝑥 = 𝜋�4 , 𝑢 = tan 4 = 1
𝜋

When 𝑥 = 0, 𝑢 = tan 0 = 0
Hence
𝜋
1 1
4
2 tan 𝑥
𝑑𝑢
� sec 𝑥𝑒 𝑑𝑥 = � sec 𝑥 𝑒 � 2 � = � 𝑒 𝑢 𝑑𝑢
2 𝑢
0 0 sec 𝑥 0
[𝑒 𝑢 ]1 1 0
= 0 =𝑒 −𝑒 =𝑒−1

NB: Once you have changed the limits to those of you, do not change back to the limits
for x.

Turn to Tutorial 6 on page 32 of your study guide. From Question 1, do all the
questions from 1.1 to 1.6.
Finding area under a curve by definite integration
The area A bounded by the curve 𝑦 = 𝑓(𝑥), the lines 𝑥 = 𝑎, 𝑥 = 𝑏, and the x-axis, can be
found from the definite integral
𝑏
𝐴 = � 𝑓(𝑥)𝑑𝑥 = [𝐹(𝑥)]𝑏𝑎 = 𝐹(𝑏) − 𝐹(𝑎)
𝑎
Example
Determine the area bounded by the curve 𝑦 = 𝑓(𝑥), the x-axis and the stated x-values in the
following cases:
a) 𝑦 = 𝑥 2 + 3, 𝑥 = 1, 𝑥 = 2

102
1
b) 𝑦 = 2 𝑥 3 + 2𝑥, 𝑥 = 1, 𝑥 = 2
c) 𝑦 = 4𝑥 3 , 𝑥 = −2, 𝑥 = −1

Solution
a)
2 2
𝑥3 23 13
𝐴=� (𝑥 2 + 3)𝑑𝑥 = � + 3𝑥� = � + 3 ∙ 3� − � + 3 ∙ 1�
1 3 1
3 3
8 1 25
= � + 9� − � + 3� =
3 3 3

Therefore,
25
𝐴= unit 2
3

NB: Do not leave out the units!


b)
2 2
𝑥3 𝑥4 24 14
𝐴 = � � + 2𝑥� 𝑑𝑥 = � + 𝑥 � = � + 22 � − � + 12 �
2
1 2 8 1
8 8
1 39
= (2 + 4) − � + 1� =
8 8

Therefore
39
𝐴= unit 2
8

c)
−1
𝐴 = � 4𝑥 3 𝑑𝑥 = [𝑥 4 ]−1 4 4
−2 = (−1) − (−2) = 1 − 16 = −15
−2

Therefore
𝐴 = 15 unit 2

NB: The reason for the negative area in this result is the fact that the area is being
evaluated over the region in which the curve is below the x-axis (you can sketch a graph
to verify). The minus sign refers only to the position of A relative to the x-axis. The
actual area is thus 15 unit2, ignoring the negative sign.

Turn to page 33 of the study guide. On Question 2, do all the questions from 2.1 to
2.5.

REFLECTION AND COMMENT


In this unit, we extended what we covered in the previous and went on to apply limits to the
integral to evaluate the definite integral. The constant of integration cancels out when
evaluating the definite integral. The value of the definite integral between a curve and the x-

103
axis and bounded by two points, say a and b, is equal to the value of the area determined by
the curve:

∫ f ( x=
) dx F ( b ) − F ( a )
b
Area =
a

where F ( x ) = ∫ f ( x ) dx. Always remember to put the appropriate units.

This officially marks the end of this content area, I hope you found as exciting as I did, I
really enjoyed myself. It is long but the foundation you gain here is necessary for your
Mathematics 2 and beyond as I said earlier on.
In the next section, we are going into the imaginary world of complex numbers and this will
be out last content area for Mathematics 1. However, it is a very important section especially
to those who are doing Electrical Engineering.

104
CONTENT AREA 5: COMPLEX NUMBERS

COMPLEX NUMBERS

Equations with complex roots

Rectangular Form of Complex Numbers

Argand Diagram

Polar Form of Complex Numbers

Exponential Form of Complex Numbers

De Moivre's Theorem

105
15 Complex Numbers: Basic Definitions; Operations and Graphical
Representation
Learning Outcomes
After completing this section, you should be able to
• Recognise j as standing for √−1 and be able to reduce powers of j to ±j or ±1
• Recognise that all complex numbers are in the form (real part) + j (imaginary part)
• Add, subtract and multiply complex numbers
• Find the complex conjugate of a complex number
• Divide complex numbers
• State the conditions for the equality of two complex numbers
• Represent a complex number in rectangular form on an Argand diagram.

15.1 Introduction
In the first content area we discussed the solving of quadratic equations. We also looked at
the quadratic formula which we can use to solve any quadratic equation. For the record, the
quadratic formula to solve the quadratic equation
𝑎𝑥 2 + 𝑏𝑥 + 𝑐 = 0 is given by
−𝑏 ± √𝑏 2 − 4𝑎𝑐
𝑥=
2𝑎

It is clear from the above equation that the nature of the roots we obtained is governed by the
term under the root sign, that is, 𝑏 2 − 4𝑎𝑐.

We have the following three types of roots depending on this value


1) 𝑏 2 − 4𝑎𝑐 > 0: In this case, the quadratic equation has two distinct real roots
2) 𝑏 2 − 4𝑎𝑐 = 0: In this case, the quadratic equation has a repeated root which is equal
𝑏
to − 2𝑎.
3) 𝑏 2 − 4𝑎𝑐 < 0: This is the case we haven’t looked at so far. The negative term results
in the square root of a negative, the solution of which does not exist in the real
number system. Graphically, the quadratic curve does not cross the x-axis hence the
roots are called imaginary roots. These roots are defined in what is called the complex
number system and this is going to form the subject of our study in this content area.

15.2 The imaginary number 𝒋


To deal with the problem that 𝑥 2 = −1 has no solution in the real number system,
mathematicians of the eighteenth century invented the “imaginary” number
𝑖 = √−1

This number 𝑖 has lots of uses in engineering. However, the letter 𝑖 is reserved for current in
electrical engineering and so an alternative letter was adopted by the engineers to counter the
conflicting symbol and thus the symbol 𝑗 = √−1 has since been adopted.

106
Just like its mathematics counterpart 𝑖, the symbol 𝑗 represents the imaginary number √−1.
And so from this moment on, we will drop the symbol 𝑖 and will be using the symbol 𝑗 for
the remainder of the course and beyond.
Powers of 𝒋
Since 𝑗 = √−1, let’s consider some powers of j.
𝑗 = √−1
𝑗 2 = −1
𝑗 = 𝑗 2 ∙ 𝑗 = −𝑗
3

𝑗4 = 𝑗2 ∙ 𝑗2 = 1
𝑗 5 = 𝑗 4𝑗 = 𝑗
𝑗 6 = 𝑗 4 𝑗 2 = −1
𝑗 = 𝑗 4 𝑗 3 = 𝑗 3 = −𝑗
7

𝑗 8 = 𝑗 4𝑗 4 = 1

A definite pattern is emerging here on the powers of 𝑗. Everytime a factor of 𝑗 4 occurs, it can
be replaced by 1 and the same holds for all powers of 𝑗 which are multiples of 4. By knowing
only the first four powers of 𝑗, we can easily simplify any power of j

Example
Simplify the following powers of 𝑗
a) 𝑗 17
b) 𝑗 38
c) 𝑗 63
d) 𝑗 75
e) 𝑗 2014

Solution
a) 𝑗 17 = 𝑗 16 𝑗 = (𝑗 4 )4 𝑗 = 𝑗
b) 𝑗 38 = 𝑗 36 𝑗 2 = −1
c) 𝑗 63 = 𝑗 60 𝑗 3 = −𝑗
d) 𝑗 75 = 𝑗 72 𝑗 3 = −𝑗
e) 𝑗 2014 = (𝑗 4 )503 𝑗 2 = 𝑗 2 = −1

Simplify the following powers of 𝑗.


42
a) 𝑗
b) 𝑗 11
c) 𝑗 112
d) 𝑗 45

Since 𝑗 = √−1, it then becomes possible to evaluate the roots of negative numbers. For
example, √−64 = √−1 × √64 = 𝑗8
√−16 = √−1 × √16 = 𝑗4

𝑗8 and 𝑗4 are called purely imaginary numbers. Just like the case for real numbers, you can
add or subtract two imaginary numbers.

107
For example
𝑗8 + 𝑗4 = 𝑗12
𝑗8 − 𝑗4 = 𝑗4

15.3 Complex Numbers


When a real number is added to an imaginary number, we get what is called a complex
number. More formally, a complex number is in the form
𝑥 + 𝑗𝑦

Where 𝑥 ∈ 𝑅 is called the imaginary part of the complex number and;


𝑦 ∈ 𝑅 is called the imaginary part of the complex number.

Complex numbers have many applications in engineering. To be able to use them, we need to
know and understand how to carry out the usual arithmetic operations.
Addition and Subtraction of Complex Numbers
The addition and subtraction of complex numbers is very straight forward, you add the real
part to the real part and the imaginary part to the imaginary part. This is synonymous to the
concept of collecting like terms in algebraic arithmetic operations.

For example, if 𝑧1 = −3 + 𝑗8 and 𝑧2 = 6 + 𝑗3, we have

𝑧1 + 𝑧2 = (−3 + 𝑗8) + (6 + 𝑗3) = −3 + 6 + 𝑗8 + 𝑗3 = 3 + 𝑗11


𝑧2 − 𝑧1 = (6 + 𝑗3) − (−3 + 𝑗8) = 6 + 𝑗3 + 3 − 𝑗8 = 6 + 3 + 𝑗3 − 𝑗8 = 9 − 𝑗5

Multiplication of Complex Numbers


The multiplication of complex numbers is distributive across the product. Remember from
elementary algebra that
(𝑎 + 𝑏)(𝑐 + 𝑑) = 𝑎𝑐 + 𝑎𝑑 + 𝑏𝑐 + 𝑏𝑑

The same approach is the same with dealing with complex numbers. However, one always
has to remember that 𝑗 2 = −1 in the simplification after clearing out the brackets. For
example, if 𝑧1 = −3 + 𝑗8 and 𝑧2 = 6 + 𝑗3, we have

𝑧1 𝑧2 = (−3 + 𝑗8)(6 + 𝑗3) = −18 − 𝑗9 + 𝑗18 + 𝑗 2 24 = −18 + 𝑗9 − 24 = −42 + 𝑗9

Example
Let 𝑧1 = 2 + 𝑗3 and 𝑧2 = 2 − 𝑗3. Evaluate 𝑧1 𝑧2 .

Solution
𝑧1 𝑧2 = (2 + 𝑗3)(2 − 𝑗3) = 4 − 𝑗6 + 𝑗6 − 𝑗 2 9 = 4 + 9 = 13

There is no mistake here and the result 13 is not a complex number but a real number. The
result is true for all products complex numbers such that 𝑧1 = 𝑥 + 𝑗𝑦 and 𝑧2 = 𝑥 − 𝑗𝑦. This
brings us to an important definition which we will use extensively in the next section on the
division of complex, that is, the complex conjugate.
Let 𝑧 = 𝑥 + 𝑗𝑦 be a complex number. The complex conjugate is defined as the complex
number 𝑧̅ = 𝑥 + 𝑗𝑦.

108
Division of Complex Numbers
Just as complex numbers can be added, subtracted and multiplied, complex numbers can be
divided as well. However, to rationalise the complex number into the form 𝑥 + 𝑗𝑦, there’s
need to multiply the numerator and the denominator by the complex conjugate of the
denominator.

Example
𝑧
If 𝑧1 = −3 + 𝑗8 and 𝑧2 = 6 + 𝑗3, evaluate 𝑧1 giving the final answer in the form 𝑧 = 𝑥 + 𝑗𝑦.
2

Solution
𝑧1 −3 + 𝑗8 (−3 + 𝑗8) (6 − 𝑗3) −18 + 𝑗9 + 𝑗48 − 𝑗 2 24
= = =
𝑧2 6 + 𝑗3 (6 + 𝑗3) (6 − 𝑗3) 45
−18 + 𝑗57 + 24 6 + 𝑗57 6 57 2 19
= = = +𝑗 = +𝑗
45 45 45 45 9 15
NB: Remember to express fractions in their lowest terms.

Turn to page 33 of the study guide. From Tutorial 7 on Question 1, do all the
questions from 1.1 to 1.5.

Equality of Complex Numbers


Let 𝑧1 = 𝑎 + 𝑗𝑏 and 𝑧2 = 𝑐 + 𝑗𝑑 be two complex numbers. We say two complex two
complex numbers are equal if and only if
𝑎 = 𝑐 (Real part)

and
𝑏 = 𝑑 (Imaginary Part)

With this in mind, this means we are able to compare any two complex numbers so long they
have been simplified and expressed in terms of 𝑗.

Example
a) Simplify
𝑗4 − 𝑗3
(7 − 𝑗)(2 + 𝑗) −
3+𝑗
b) Solve for x and y if
(1 + 𝑗3)2
= 𝑥 + 𝑦𝑗 5
2𝑗 2 + 𝑗
Solution
a)
𝑗4 − 𝑗3
(7 − 𝑗)(2 + 𝑗) −
3+𝑗
1+𝑗
= 14 + 𝑗7 − 𝑗2 − 𝑗 2 −
3+𝑗
(1 + 𝑗) (3 − 𝑗)
= 15 + 𝑗5 −
(3 + 𝑗) (3 − 𝑗)

109
3 − 𝑗 + 𝑗3 − 𝑗 2
= 15 + 𝑗5 −
10
4 + 𝑗2
= 15 + 𝑗5 −
10
150 + 𝑗50 − 4 − 𝑗2
=
10
146 + 𝑗48
=
10
146 48
= +𝑗
10 10
73 24
= +𝑗
5 5

COMMENT: With practice, you can compress some of the steps depending on your
understanding of the material.
b)
(1 + 𝑗3)2
2
= 𝑥 + 𝑦𝑗 5
2𝑗 + 𝑗

COMMENT: To solve this question, you first simplify both sides into the form 𝑥 + 𝑦𝑗 and
compare the real and the imaginary parts. This will result in a pair of simultaneous equations
in x and y.
1 + 𝑗6 + 𝑗 2 9
⇒ = 𝑥 + 𝑗𝑦
−2 + 𝑗
−8 + 𝑗6
⇒ = 𝑥 + 𝑗𝑦
−2 + 𝑗
(−8 + 𝑗6) (−2 − 𝑗)
⇒ = 𝑥 + 𝑗𝑦
(−2 + 𝑗) (−2 − 𝑗)
22 − 𝑗4
⇒ = 𝑥 + 𝑗𝑦
5
⇒ 22 − 𝑗4 = 5𝑥 + 𝑗5𝑦

Equating the real parts, we have


22 = 5𝑥
22
⇒ 𝑥=
5

Equating the imaginary parts, we have


4
−4 = 5𝑦 ⇒ − = 𝑦
5
4
∴ 𝑦=−
5

Turn to page 34 of the study guide. On Question 2, do questions 2.1 to 2.3.

110
15.4 Graphical representation of Complex Numbers
The graphical representation of complex numbers is the same as that for the normal Cartesian
plane. The x-axis represents the real part while the y-axis represents the imaginary part of the
complex number. When a Cartesian plane is used in this way to represent complex numbers,
it becomes known as the Complex plane or the Argand diagram. From a geometric or
graphical point of view, the complex number 𝑧 = 𝑥 + 𝑗𝑦 is identical to the point (𝑥; 𝑦) in the
plane. An example of an Argand diagram with some complex numbers is shown below.

The plotting of complex numbers on the Argand diagram leads us to the next topic where we
will be introducing alternative forms of the complex numbers like the polar form and the
exponential which make use of the argument and the modulus of the complex number.

REFLECTION AND COMMENT


A complex number is a number z of the form
z= x + jy
where x and y are real numbers and j= −1. x is called the real part and y is called the
imaginary part of a complex number. The form z= x + jy is called the rectangular form of a
complex number. Two complex numbers are equal if and only if the real part and the
imaginary part of the two respective complex numbers are equal.
Even though complex numbers in themselves are imaginary, they have many useful
properties in engineering and mathematics. You can add, subtract, multiply and divide two
complex numbers.
Complex numbers can be easily plotted on a graph, called an Argand diagram in the case of a
complex number. The coordinate for a complex number z= x + jy are simply ( x, y ) .

In the next unit, we go on to explore other alternative forms of complex numbers like the
polar form and exponential form; as well as the application of De Moivre’s theorem to
determine the roots of a complex number.

111
16 Complex Numbers: Polar, Exponential Forms and De Moivre’s
Theorem
Learning Outcomes
After completing this section, you should be able to
• Convert a complex number from rectangular form into the polar and exponential
forms
• Use De Moivre’s theorem to raise a complex number to a power
• Use De Moivre’s theorem to find the nth root of a complex number

16.1 Polar and Exponential Form of a Complex Number


Let 𝑧 = 𝑥 + 𝑗𝑦 be a complex number. This way of expressing a complex number is called the
Cartesian or rectangular form. We now want to explore other ways of expressing complex
numbers but we start by defining some important terms first.
The modulus of the complex number is defined as
|𝑧| = �𝑥 2 + 𝑦 2

and the argument of the complex number 𝜃 is defined as


𝑦
𝜃 = tan−1
𝑥

The quadrant in which θ falls comes from the Argand diagram; as there are always two
possible values, whether tan is positive or negative. Since you have already covered the
general solution, you already know that the value of θ is not unique as the value repeats every
2𝜋 radians or 360°. However, the value of the argument we are looking for should be in the
range −𝜋 ≤ 𝜃 ≤ 𝜋 or −180° ≤ 𝜃 ≤ 180°.
NB: Degrees are commonly used in this course but solutions in radians are permissible.
You should choose whether you want to use degrees or radians and stick to your desired
units.

|𝑧| is commonly called 𝑟, hence from your knowledge of trigonometry,


𝑥 = 𝑟 cos 𝜃
𝑦 = 𝑟 sin 𝜃

112
But we already know that 𝑧 = 𝑥 + 𝑗𝑦, therefore
𝑧 = 𝑥 + 𝑗𝑦 = 𝑟 cos 𝜃 + 𝑗 𝑟 sin 𝜃
∴ 𝒛 = 𝒓(𝐜𝐨𝐬 𝜽 + 𝒋 𝐬𝐢𝐧 𝜽)

The above form of z is what is called, the Polar Form of a complex number.

Example
Express each of the following in polar form
a) −1 + 𝑗√3
b) 3 − 𝑗3

Solution
a) To get the modulus, we say
2
𝑟 = �𝑥 2 + 𝑦 2 = �(−1)2 + �√3� = √1 + 3 = 2

NB: The value of r is always positive; ignore the negative value of the square root. You
will be penalised if you give the final answer for the modulus as 𝒓 = ±𝟐.
To get the modulus, we say
√3
θ = tan−1 � � = −60° or 120°
−1

Because x is negative and y is positive, θ is clearly in the 2nd quadrant. You can do back to
the trigonometry solution to read on the quadrants. So the argument in this case is
𝜃 = 120°
Hence
−1 + 𝑗√3 = 2(cos 120° + 𝑗 sin 120°)

b) I will keep explanations to the minimum in this one since all the points have been
highlighted in (a).
𝑟 = �(3)2 + (−3)2 = √9 + 9 = √18 = √2 × 9 = 3√2

NB: Do not use a calculator; leave the answer in surd form unless specifically instructed
otherwise in the question.
−3
𝜃 = tan−1 � � = −45°
3

Hence
3 − 𝑗3 = 3√2(cos(−45°) + 𝑗 sin(−45°))
There is yet another form of expressing a complex that you will need to be familiar with in
this course and that is the exponential form. The exponential form is based on the power
series of the exponential function 𝑒 𝑥 but the details are beyond the scope of this course.
We have already seen how to get the modulus and argument of a complex number to convert
the complex number to polar form. Using these two values, we have
𝑥 + 𝑗𝑦 = 𝑟(cos 𝜃 + 𝑗 sin 𝜃) = 𝒓𝒆𝒋𝜽

𝑟𝑒 𝑗𝜃 is thus called the exponential form of a complex number.

113
For example, from the previous examples, we have
a)
−1 + 𝑗√3 = 2(cos 120° + 𝑗 sin 120°) = 2𝑒 𝑗120°
b)
3 − 𝑗3 = 3√2(cos(−45°) + 𝑗 sin(−45°)) = 3√2𝑒 −𝑗45°

Turn to page 34 of the study guide. From Question 4, do all the questions from 4.1
to 4.5.

The process of converting back from polar/exponential form to rectangular form is rather
straight forward. You start by evaluating the cosine and sine functions then multiply
throughout by the modulus as indicated in the example below.

Example
Express 2√2[cos(−45°) + 𝑗 sin(−45°)] in rectangular form.

Solution
1 1
cos(−45°) = , sin(−45°) = −
√2 √2

Hence
1 1
2√2[cos(−45°) + 𝑗 sin(−45°)] = 2√2 � −𝑗 � = 2 − 𝑗2
√2 √2

NB: I strongly advise the use of a Casio scientific calculator for these types of questions as
the Casio scientific calculator will give you solutions is surd form.

Turn to page 35 of the study guide. From Question 5, do questions 5.1 to 5.3.

16.2 Multiplication and Division of Complex Numbers in Polar Form


Let 𝑧1 = 𝑎 + 𝑗𝑏 = 𝑟1 (cos 𝜃1 + 𝑗 sin 𝜃1 ) and 𝑧2 = 𝑐 + 𝑗𝑑 = 𝑟2 (cos 𝜃2 + 𝑗 sin 𝜃2 ).
We have the following important results for the product and quotient using the polar form of
complex numbers
𝑧1 ∙ 𝑧2 = 𝑟1 𝑟2 [cos(𝜃1 + 𝜃2 ) + 𝑗 sin(𝜃1 + 𝜃2 )] = 𝑟1 𝑟2 𝑒 𝑗(𝜃1 +𝜃2 )
𝑧1 𝑟1 𝑟1
= [cos(𝜃1 − 𝜃2 ) + 𝑗 sin(𝜃1 − 𝜃2 )] = 𝑒 𝑗(𝜃1 −𝜃2 )
𝑧2 𝑟2 𝑟2

Clearly, the polar form makes it so easy and is very convenient when multiplying or dividing
two (or more) complex numbers.

114
Turn to page 35 of the study guide and do Question 6.

16.3 De Moivre’s Theorem


De Moivre’s Theorem states that;
If 𝑧 = 𝑟(cos 𝜃 + 𝑗 sin 𝜃), then 𝑧 𝑛 = 𝑟 𝑛 [cos(𝑛𝜃) + 𝑗 sin(𝑛𝜃)] for every natural number n.

Example
If 𝑧 = 3 − 𝑗3, evaluate 𝑧 4 leaving your answer in rectangular form.

Solution
𝑟�32 + (−3)2 = √18 = 3√2
−3
𝜃 = tan−1 � � = −45°
3
∴ 𝑧 = 3 − 𝑗3 = 3√2[cos(−45°) + 𝑗 sin(−45°)]
By De Moivre’s Theorem,
4
⇒ 𝑧 4 = �3√2� [cos(−45° × 4) + 𝑗 sin(−45° × 4)]
⇒ 𝑧 4 = 324[cos(−360°) + 𝑗 sin(−360°)]
⇒ 𝑧4 = 324(cos 0° + 𝑗 sin 0°)
⇒ 𝑧 4 = 324 = 324 + 𝑗0

NB: Remember, even if the final answer is a real number, you have to express it in
complex rectangular form as indicated above.

Turn to page 36 of the study guide. On Question 7, do questions 7.1 to 7.3.

REFLECTION AND COMMENT


In the unit, we have seen that there are 3 equivalent ways of expressing a complex number:
• rectangular form: z= x + jy;
y
• polar form:= z r ( cos θ + j sin θ ) , where=r x 2 + y 2 and tan θ = ; and
x
• the exponential form: z = re jθ . The exponential form is closely related to the polar
form.
The polar form and the exponential form of a complex number make it more convenient to
perform multiplication and division between two complex numbers:
z1 r1e jθ1 r1 j (θ1 −θ2 )
= = e ;
z2 r2 e jθ2 r2
z1 ⋅ z2= r1e jθ1 ⋅ r2 e jθ2= r1r2 e j (θ1 +θ2 )
To evaluate all the nth roots of a complex number z, we use De Moivre’s theorem to say
1 1   θ + 2π k   θ + 2π k  
= z n r n  cos   + j sin   ;
  n   n  
= k 0,1, , n − 1

115
We have now come to the end of the course Mathematics 1 but certainly, this is not
the end of learning. It’s now time you quickly revise this whole tutorial guide and also get to
sample a few past examination papers which you will find on Blackboard. Do not just look at
the answers, but also get to understand the structure of the paper with particular emphasis on
how much each section contributes and also to gauge your exam time management skills.
After that, relax! And let’s meet at Seme Hall.
I wish you all the best in you’re the rest of your studies.

116
REFERENCES

BOSTOCK, L. & CHANDLER, S. 1995. Core Maths for A Level, Cheltenham, United Kingdom,
Stanley Thornes Ltd.
CHANNON, J. B., SMITH, A. M., HEAD, H. C., MACRAE, M. F. & CHASAKARA, A. A. 2009. New
General Mathematics 3, Essex, United Kingdom, Longman Group UK Limited.
STROUD, K. A. & BOOTH, D. J. 2001. Engineering Mathematics, Wiltshire, Great Britain,
PALGRAVE.

117

You might also like